You are on page 1of 74

Are you ready for Beast Academy 1A?

Beginner
Before beginning readers may A student
Beast Academy 1A, need lots of help ready for Beast
students should be using Beast Academy 1A should
able to count and write Academy 1A. be able to answer at
numbers to at least 20, least 12 of the 16
preferably to 100. problems below
correctly.

Step 1. The student should try to answer every question with reading help from a parent if
needed.
Step 2. Check the student’s answers using the solutions at the end of this document.
Step 3. The student should be given a second chance on problems 5-16 if answered incorrectly.

Circle the box that has more dots.

1. or 2. or

3. or 4. or

Write the number of dots in each group below.

5. ______ 6. ______ 7. ______

© 2021, AoPS Incorporated.


Are you ready for Beast Academy 1A?

Starting with the smallest number, draw straight lines to connect the
numbered dots in order.
8. 9.
12 16 17
7
9 12 13

6 10 8 9

11 10
11 5
8 15 14

Fill in the missing numbers in each number pattern below.

10. 7 8 9 11 13 15 17 18

11. 20 19 18 14 10

12. 8 9 10 14 15 16

Fill each blank with a number to complete the statement.

13. Two more than five is _____. 14. Four more than seven is _____.

15. Three less than eight is _____. 16. Six more than nine is _____.

© 2021, AoPS Incorporated.


Are you ready for Beast Academy 1A?
Solutions
1. Five is more then four. 9. Connecting the dots creates the spiral pattern below.
16 17
or
12 13

2. Seven is more than six. 8 9

or 11 10

15 14
3. Seven is more than six. If your child got this one wrong,
they are not alone in thinking that the longer row of dots 10. The pattern counts up by 1’s from 7 to 18.
has more!
7 8 9 10 11 12 13 14 15 16 17 18

or
11. The pattern counts down by 1’s from 20 to 9.

20 19 18 17 16 15 14 13 12 11 10 9

4. If your student solved this one by counting 25 dots and


24 dots, ask if they see a better way to solve it without 12. The pattern counts up by 1’s from 5 to 16.
counting. 5 6 7 8 9 10 11 12 13 14 15 16

or
13. Two more than five is 7.
For this and the following problems, any method is fine.
Counting up on fingers, drawing dots or sticks, gathering
groups of small objects like Cheerios... whatever method
5. There are 7 dots. a student uses to get the right answer is great as long as
6. There are 12 dots. they did it on their own.
7. There are 14 dots. Students do not need to think of this as 5+2 or 2+5 (but
8. It’s fine if your student’s lines are not as straight as it’s fine if they do). Students beginning in 1A should have
the ones below. However, students will do some dot a good sense of how numbers within 20 relate.
connecting in Chapter 2 of 1A, so some fine motor skills 14. Four more than seven is 11.
are required. 15. Three less than eight is 5.
12
16. Six more than nine is 15.
7
9

6 10

11 5
8
(It’s also fine if your child couldn’t help themselves and
connected 12 to 5 to complete the star, as long as they
know that 5 doesn’t come after 12.)
Are you ready for Beast Academy 1B?
Before Beginner
beginning Beast readers may A student
Academy 1B, students need lots of help ready for Beast
should be able to count using Beast Academy 1B should
and compare numbers Academy 1B. be able to answer at
within 100 and recognize least 12 of the 17
some basic shapes and problems below
their properties. correctly.

Step 1. The student should try to answer every question with reading help from a parent if
needed.
Step 2. Check the student’s answers using the solutions at the end of this document.
Step 3. The student should be given a second chance on problems answered incorrectly.

Fill in the numbers of dots for each table below.

1. Filled dots  = ________ 4. Filled dots  = ________

2. Empty dots  = ________ 5. Empty dots  = ________

3. Total dots = ________ 6. Total dots = ________

Fill each circle below with <, >, or = to compare the amounts below.

7. 8. 9. 54 45
© 2022, AoPS Incorporated.
Are you ready for Beast Academy 1B?

Trace the lines inside each shape below to show how it could be split
into two pieces that are the same shape and size.

10. 11. 12.

Fill each blank below with a digit (0, 1, 2, 3, 4, 5, 6, 7, 8, or 9) so that the


numbers are in order from smallest to largest.

13. 35 < _5 < 50 14. 73 < _1 < _0

15. _8 < _7 < 32 16. 49 < _8 < 5_

Connect dots on the right to draw the shape on the left.


17.

© 2022, AoPS Incorporated.


Are you ready for Beast Academy 1B?
Solutions
1. Filled: 43 13. 35 is less than 45 which is less than 50.
2. Unfilled: 27
3. Total: 70 35 < 4
_5 < 50
4. Filled: 34
5. Unfilled: 33 14. 73 is less than 81 which is less than 90.
6. Total: 67
7. 9 is less than 10. 73 < 8
_1 < 9
_0
15. 18 is less than 27 which is less than 32.

8. 12 is equal to 12. 1_8 < 2


_7 < 32
16. 49 is less than 58 which is less than 59.

9. 54 is greater than 45. 49 < 5


_8 < 5_
9
54 45 17. We connect the dots as shown below.

10. We can split the shape into two pieces that are the same
shape and size as shown.

11. We can split the shape into two pieces that are the same
shape and size as shown.

12. We can split the shape into two pieces that are the same
shape and size as shown.
Are you ready for Beast Academy 1C?
Beginner A student
Before beginning readers may
Beast Academy 1C, ready for Beast
need lots of help Academy 1C should
students should be using Beast
able to add and subtract be able to answer at
Academy 1C. least 12 of the 17
numbers within 100
using a variety of problems below
strategies. correctly.

Step 1. The student should try to answer every question with reading help from a parent if
needed.
Step 2. Check the student’s answers using the solutions at the end of this document.
Step 3. The student should be given a second chance on problems answered incorrectly.

Fill in the blanks to make the equations below true.

1. 8+8= 2. 15−6= 3. =30+10

4. 7+ =20 5. 59− =53 6. +7=80

7. 32−9=32−2− 8. 25+14=25+10+

9. 7+29+3=10+ 10. 38−2−2−2=38−

© 2022, AoPS Incorporated.


Are you ready for Beast Academy 1C?
Cross out numbers in each grid so that the remaining numbers in every
row and column add up to the target sum as shown in the example.

Target: 10 Target: 12 Target: 20


Ex. 5 7 3 11. 3 9 5 12. 15 10 10
5 6 5 2 3 7 15 5 10
5 3 2 7 12 5 5 5 10

Target: 15 Target: 18 Target: 16


13. 8 7 7 14. 6 6 12 15. 11 11 5
8 8 7 9 9 6 5 5 11
7 6 8 9 3 6 5 11 5

Fill the empty squares below with numbers so that the equations from
left-to-right and top-to-bottom are all correct.

16. 30 + 5 = 17. − 7 =
+ + + − − −
+ = 41 − =
= = = = = =
40 + 12 = 8 − = 3

© 2022, AoPS Incorporated.


Are you ready for Beast Academy 1C?
Solutions
Target: 18
1. 8+8= 16
2. 15−6= 9 14. 6 6 12
3. 40 =30+10 9 9 6
4. 7+ 13 =20
5. 59− 6 =53 9 3 6
6. 73 +7=80
Target: 16
7. 32−9=32−2− 7
8. 25+14=25+10+ 4 15. 11 11 5
9. 7+29+3=10+ 29 5 5 11
10. 38−2−2−2=38− 6
5 11 5
Ideally, students should be recognizing relationships
between numbers in problems 7 through 10. For 16. 30 + 5 = 35
example, students should recognize that subtracting three
2’s in problem ten is the same as subtracting 6 (without + + +
ever computing 38−2−2−2=32).
Students who are not spotting these relationships might 10 + 7 = 17
benefit from starting in Beast Academy 1B, which focuses
on a variety of addition and subtraction strategies and = = =
helps build number sense for young students.
40 + 12 = 52
Target: 12

11. 3 9 5 49 − 7 = 42
17.
2 3 7
− − −
7 12 5
41 − 2 = 39
Target: 20 = = =
12. 15 10 10
8 − 5 = 3
15 5 10
5 5 10

Target: 15

13. 8 7 7
8 8 7
7 6 8
Are you ready for Beast Academy 1D?
Beginner
Before beginning readers may A student
Beast Academy 1D, need lots of help ready for Beast
students should be using Beast Academy 1D should
able to add and subtract Academy 1D. be able to answer at
numbers within 100 least 15 of the 21
using a variety of problems below
strategies. correctly.

Step 1. The student should try to answer every question with reading help from a parent if
needed.
Step 2. Check the student’s answers using the solutions at the end of this document.
Step 3. The student should be given a second chance on problems answered incorrectly.

Fill in the blanks to make the equations below true.

1. 40+20= 2. 50+30=

3. 70−20= 4. 90−30=

5. 32+20= 6. 47+ =77

7. 20+34=50+ 8. 39+37=40+

9. 7+9+ =20 10. 56−29= −30

© 2022, AoPS Incorporated.


Are you ready for Beast Academy 1D?

36 40 45 50 60 63

Use the number line above to help you find each distance below.

11. From 36 to 45 is _______ units. 12. From 45 to 63 is _______ units.

13. From 45 to 60 is _______ units. 14. From 36 to 63 is _______ units.

Fill the empty boxes to complete each pattern below.

+10 +10 +10 +10 +10 +10 +10 +10

15. 23 63
+5 +5 +5 +5 +5 +5 +5 +5

16. 25 30
+3 +3 +3 +3 +3 +3 +3 +3

17. 54 57

Place a <, >, or = in each circle below.

18. 10+20 11+22 19. 38+39 60+16

20. 35+53 33+55 21. 23+24+25 24+25+26

© 2022, AoPS Incorporated.


Are you ready for Beast Academy 1D?
Solutions
+10 +10 +10 +10 +10 +10 +10 +10
1. 40+20= 60
15. 3 13 23 33 43 53 63 73 83
2. 50+30= 80
3. 70−20= 50
+5 +5 +5 +5 +5 +5 +5 +5
4. 90−30= 60
5. 32+20= 52 16. 15 20 25 30 35 40 45 50 55
6. 47+ 30 =77
+3 +3 +3 +3 +3 +3 +3 +3
7. 20+34=50+ 4 (Add the tens first: 20+30=50.)
8. 39+37=40+ 36 17. 42 45 48 51 54 57 60 63 66
(Think of 39+37 as apples in two baskets. We can take
one apple from the basket of 37 and put it in the basket of

9.
39 without changing the total number of apples.
So, 39+37=40+36.)
7+9+ 4 =20
18. 10+20
< 11+22
Each number on the right is larger than the number in its
place on the left.
10. 56−29= 57 −30
We can think of subtraction as the distance between two
numbers on the number line.
The distance between 56 and 29 is the same as the
19. 38+39
> 60+16
On the left, 30+30=60, and 8+9=17.
distance between 57 and 30. 60+17>60+16.

11. From 36 to 45 is 9 units.


+4 +5
20. 35+53
= 33+55
On both sides, the tens add to 80 and the ones add to 8.
(Both sides equal 88.)
36 40 45 50 60 63

12. From 45 to 63 is 18 units.


21. 23+24+25
< 24+25+26
Each number on the right is one more than the number in
+5 +10 +3 the same position on the left.
—or—
36 40 45 50 60 63 Both sides include 24+25. But, the left side has a 23 and
the right side has a 26. So, 23+24+25 < 24+25+26.
13. From 45 to 60 is 15 units.
+5 +10

36 40 45 50 60 63

14. From 36 to 63 is 27 units.


+4 +10 +10 +3

36 40 45 50 60 63
Are you ready for Beast Academy 2A?

Before beginning A student


Beast Academy 2A, a ready for Beast
student should be able The student Academy 2A should
to count beyond 100 by should also know be able to answer at
1’s, 5’s, and 10’s. basic shapes, be able to least 13 of the 17
add and subtract numbers problems below
from 1 to 20, and be able correctly.
to solve simple word
problems.

Step 1. The student should try to answer every question without a calculator and without help.
Step 2. Check the student’s answers using the solutions at the end of this document.
Step 3. The student should be given a second chance on problems answered incorrectly.

Fill the blanks to complete each counting pattern below.

1. 1, 2, 3, 4, ____, ____, ____, 8, ____, ____, ____

2. ____, ____, ____, 22, 23, ____, 25, 26, ____, ____, 29, ____

3. 10, 20, 30, ____, 50, 60, ____, ____, ____, 100

4. 5, 10, 15, 20, ____, ____, ____, ____, ____, 50

5. ____, ____, 75, 80, 85, ____, 95, _____

Answer each question about the shapes shown below.

6. How many circles are there? 6. ________

7. How many more circles are there than squares? 7. ________


© 2017, AoPS Incorporated.
Are you ready for Beast Academy 2A?

Fill the blanks in each problem below.

8. 2+7=_____ 9. 8+12=_____ 10. 4+_____=11

11. 9−6=_____ 12. 17−8=_____ 13. _____−13=6

Solve each problem below.

14. Mike puts 6 pennies in an empty jar. Nelly puts 7 more 14. ________
pennies in the jar. Orson takes all of the pennies out of
the jar. How many pennies does Orson take?

15. There are 3 boys and 6 girls in Mr. Hai’s class. How many 15. ________
pencils are needed to give each student in Mr. Hai’s class
two pencils?

16. Fill each blank with the same number in the addition problem below.

_____+_____+_____=18

17. Fill the empty squares below so that the numbers in each row (left to right)
and each column (top to bottom) add up to 20.

8 4
5
11

© 2017, AoPS Incorporated.


Are you ready for Beast Academy 2A?
Solutions
1. Counting by ones, we have 17. In the top row, we have 8 and 4. We know 8+4=12.
1, 2, 3, 4, 5, 6, 7, 8, 9, 10, 11. Since 12+8=20, the missing number in the top row is 8.

2. Counting by ones, we have


8 8 4
19, 20, 21, 22, 23, 24, 25, 26, 27, 28, 29, 30.
3. Counting by tens, we have 5
10, 20, 30, 40, 50, 60, 70, 80, 90, 100. 11
4. Counting by fives, we have
5, 10, 15, 20, 25, 30, 35, 40, 45, 50. In the left column, we have 8 and 5. We know 8+5=13.
Since 13+7=20, the missing number in the left column is 7.
5. Counting by fives, we have
65, 70, 75, 80, 85, 90, 95, 100. 8 8 4
6. We see two groups of 5 circles, plus a circle at each end.
5

7 11

Two groups of 5 is 10, and plus 2 more is 12. So, there In the bottom row, we have 7 and 11. We know 7+11=18.
are 12 circles. Since 18+2=20, the missing number in the bottom row is 2.
7. In the previous problem, we counted 12 circles. We also
count 9 squares. 8 8 4

7 2 11
So, there are 12−9=3 more circles than squares.
In the middle column, we have 8 and 2. We know 8+2=10.
8. 2+7=9. Since 10+10=20, the missing number in the middle
9. 8+12=20. column is 10.
10. 4+7=11.
8 8 4
11. 9−6=3.
12. 17−8=9. 5 10
13. 19−13=6. 7 2 11
14. Together, Mike and Nelly put 6+7=13 pennies into the
jar. Orson takes all of the pennies. So, Orson takes 13 In the middle row, we have 5 and 10. We know 5+10=15.
pennies. Since 15+5=20, the missing number in the middle row is 5.
15. All together, there are 3+6=9 students in Mr. Hai’s class.
Each of the 9 students gets 2 pencils. So, we add nine 2’s 8 8 4
to get the total number of pencils needed:
5 10 5
2+2+2+2+2+2+2+2+2=18.
or 7 2 11
Adding nine 2’s is the same as adding two 9’s. So,
9+9=18 pencils are needed.
16. We start by trying a number that is easy to add, such as
5. Adding three 5’s gives
5+5+5=15.
15 is too small, so we try adding a larger number. Adding
three 6’s gives
6+6+6=18.
This is the correct result! So, we fill each blank with 6.

© 2017, AoPS Incorporated.


Are you ready for Beast Academy 2B?

Before beginning A student


Beast Academy 2B, ready for Beast
students should be fluent Academy 2B should
with place value, number Students should also be able to answer at
lines, and the comparison be able to add 2-digit least 14 of the 18
symbols < and >. and 3-digit numbers problems below
without stacking by using correctly.
place value or other
mental strategies.

Step 1. The student should try to answer every question without a calculator and without help.
Step 2. Check the student’s answers using the solutions at the end of this document.
Step 3. The student should be given a second chance on problems answered incorrectly.

1. Circle every number below whose ones digit is larger than its hundreds digit.

675 567 756 576 765 657

Fill in the blanks to complete the statements below.

2. 4 tens and 13 ones is the same as 5 tens and _____ ones.

3. 34 tens is the same as _____ hundreds and 4 tens.

Label the missing numbers on each number line below.

4.
40 45

5.
100 200

© 2018, AoPS Incorporated.


Are you ready for Beast Academy 2B?

Fill each circle below with < or >.

6. 87 78 7. 60+6 70−7 8. 29 ones 3 tens

Fill in the blanks to complete each sum below.

9. 43+56=_____ 10. 215+340=_____ 11. 27+44=_____

12. 397+65=_____ 13. 626+99=_____ 14. 444+ _____=876

Solve each problem below.

15. What is the sum of every whole number from 21 to 29? 15. ________

16. Juan doubles a number, then doubles the result and gets 140. 16. ________
What number did Juan start with?

17. Fill each blank below with a number using only the digits 5 and 8.

55 < ______ < 85 < 88 < ______ < 558 < ______ < ______ < 855

18. In the Sum Pyramid below, each block contains the sum of the two numbers
below it. Fill each empty block with the correct number.

500

195

190

© 2018, AoPS Incorporated.


Are you ready for Beast Academy 2B?
Solutions
1. For any three-digit number, the hundreds digit is the 9. 43+56=99.
leftmost digit, followed by the tens digit, then the ones 10. 215+340=555.
digit.
11. 27+44=71.
ds ns es 12. 397+65 is the same as 397+3+62.
dre Te On
Hun
+3 +62
We circle each number below whose ones digit is larger
than its hundreds digit. 397 400 462
675 567 756 576 765 657 So, 397+65=462.
13. Adding 99 is the same as adding 100, then taking away 1.
2. We can regroup 13 ones to make 1 ten and 3 ones. So, 626+99=626+100−1.
So, 4 tens and 13 ones is the same as 5 tens and 3 ones. 626+100=726 and 726−1=725.
So, 626+99=725.
14. To get from 444 to 876, we need to add 4 hundreds,
= 3 tens, and 2 ones.
So, we have 444+432=876.
15. We can add numbers in any order. So, adding 21 through
3. We can regroup 10 tens to make 1 hundred. 29 is easiest if we pair numbers whose sum is 50. We
can make 4 of these pairs.

= 21+29=50
22+28=50
23+27=50
Since 34=10+10+10+4, we can regroup 34 tens to 24+26=50
make 3 hundreds, with 4 tens left over.
So, 34 tens is the same as 3 hundreds and 4 tens. 25 is the only number we didn’t pair with another number.
So, the sum of every whole number from 21 to 29 is the
4. The tick marks on this number line count by ones. So, we same as the sum of four 50’s and one 25.
label the missing numbers as shown.
50+50+50+50+25
40 45 We add these numbers to get 225.
37 43 49
16. Since 70+70=140, the number we double to get 140 is 70.
Since 35+35=70, the number we double to get 70 is 35.
5. The tick marks on this number line count by tens. So, we
label the missing numbers as shown. Doubling 35, then doubling the result gives 140.
So, the number Juan started with is 35.
100 200 17. We fill the blanks as shown below.
80 130 170
55 < 58 < 85 < 88 < 555 < 558 < 585 < 588 < 855
6. 87 is greater than 78. So, we fill the circle as shown.
18. In the middle-left block, we have 305 +195=500.
87 > 78
In the bottom-middle block, we have 190+ 115 = 305 .
7. 60+6=66 and 70−7=63. Since 66 is greater than 63, In the bottom-right block, we have 115 + 80 =195.
we fill the circle as shown.
60+6 > 70−7 500

8. 29 ones is the same as 29, and 3 tens is the same as 30. 305 195
Since 29 is less than 30, we fill the circle as shown. 190 115 80
29 ones < 3 tens

© 2018, AoPS Incorporated.


Are you ready for Beast Academy 2C?

Before beginning A student


Beast Academy 2C, ready for Beast
Students should Academy 2C should
students should understand
also know how to be able to answer at
the basics of addition and
guess-and-check, least 12 of the 16
subtraction, parentheses,
work backwards, and problems below
symbols that stand for
draw pictures to solve correctly.
numbers, and simple
challenging problems.
equations.

Step 1. The student should try to answer every question without a calculator and without help.
Step 2. Check the student’s answers using the solutions at the end of this document.
Step 3. The student should be given a second chance on problems answered incorrectly.

Evaluate each expression below.

1. 87−35=_____ 2. 975−444=_____ 3. 555−99=_____

4. 431−350=_____ 5. 131−84−16=_____ 6. 232−75−32=_____

Solve each problem below.


7. What is 45−(12+3)? 7. ________

8. Place one pair of parentheses in the equation below to make it true.

16 − 8 − 4 − 2 = 14

9. Fill each blank below with + or − to make the equation true.

45 7 2 8=44

© 2018, AoPS Incorporated.


Are you ready for Beast Academy 2C?
Solve each problem below.

10. Evaluate the expression below when =9 and =13.

+ −5− =_____

11. Pip makes cookies. Burt makes twice as many cookies as Pip. Circle the
expression below that gives the total number of cookies Pip and Burt make.

+2 +3 + + +2 + +

Find the value of the symbol in each equation below.

12. +12=35 13. + −1=15

=_______ =_______

Solve each problem below.

14. One hundred monsters stand in a line. Arg is 15th in line. Burg 14. ________
is 25th in line. How many monsters are between Arg and Burg?

15. Each day, the number of leaves on a Magic Bush doubles. On 15. ________
Friday, a Magic Bush had 96 leaves. How many leaves did the
Magic Bush have on Monday, earlier that week?

16. Matt has 3 more pennies than Kim. Together, they have 29 16. ________
pennies. How many pennies does Kim have?

© 2018, AoPS Incorporated.


Are you ready for Beast Academy 2C?
Solutions
1. We subtract by place value. 11. Pip makes cookies. Since Burt makes twice as many
8 tens minus 3 tens is 5 tens. cookies as Pip, Burt makes + cookies.
7 ones minus 5 ones is 2 ones. So, Pip and Burt make + + cookies all together.
5 tens and 2 ones is 52. So, 87−35=52.
+2 +3 + + +2 + +
2. We subtract by place value.
9 hundreds minus 4 hundreds is 5 hundreds. 12. Since 23+12=35, we have =23.
7 tens minus 4 tens is 3 tens. 13. We subtract 1 from + to get 15. So, + is 1 more
5 ones minus 4 ones is 1 one. than 15, which is 16. Since 8+8=16, we have =8.
5 hundreds, 3 tens, and 1 one is 531. We replace with 8 to check our answer: 8+8−1=15. ü
So, 975−444=531. 14. To find the number of monsters between Arg and Burg,
3. Subtracting 99 is the same as subtracting 100, then we draw a “picture” of monsters 15 to 25 and circle the
adding 1. monsters between Arg and Burg.
So, 555−99 is the same as 555−100+1=455+1=456.
Arg Burg
4. To subtract 431−350, we can count up. 15 16 17 18 19 20 21 22 23 24 25
400 is 50 more than 350.
431 is 31 more than 400. All together, we count 9 monsters between Arg and Burg.
+50 +31 15. We work backwards. On Friday, the Magic Bush had 96
leaves, which is twice as many leaves as it had the day
before.
350 400 431 Since 48+48=96, the Magic Bush had 48 leaves on
Thursday.
So, 431 is 50+31=81 more than 350.
This means that 431−350=81. Since 24+24=48, the Magic Bush had 24 leaves on
Wednesday.
5. Subtracting 84, then subtracting 16 more is the same as
subtracting 84+16=100 all at once. So, we have Since 12+12=24, the Magic Bush had 12 leaves on
Tuesday.
131−84−16=131−100
=31. Since 6+6=12, the Magic Bush had 6 leaves on
Monday.
6. Subtracting 75, then subtracting 32 is the same as 16. We guess the number of pennies Kim has, then check
subtracting 32, then subtracting 75. So, we have the result.
232−75−32=232−32−75 If Kim has 10 pennies, then Matt has 10+3=13 pennies,
=200−75 and together they have 10+13=23 pennies. Not enough!
=125.
If Kim has 15 pennies, then Matt has 15+3=18 pennies,
7. We evaluate what’s in parentheses first. and together they have 15+18=31 pennies. Too many!
So, 45−(12+3)=45−15=30.
If Kim has 13 pennies, then Matt has 13+3=16 pennies,
8. We place the parentheses as shown below to make a and together they have 13+16=29 pennies. Got it!
true equation.
So, Kim has 13 pennies.
16−(8−4−2)=14
9. Starting with 45, we must subtract 1 more than we add to
get 44. Subtracting 7 then subtracting 2 subtracts a total
of 9, which is 1 more than 8.
So, we subtract 7 and 2, and add 8.
45 − 7 − 2 + 8=44
10. We replace the with 9 and each with 13. Then, we
evaluate the expression as shown below.
+ −5− =13+13−5−9
=26−5−9
=21−9
=12.

© 2018, AoPS Incorporated.


Are you ready for Beast Academy 2D?
A student
Before beginning ready for Beast
Beast Academy 2D, Students
Academy 2D should
students should know should understand
be able to answer at
how to measure and place value and be
least 10 of the 13
compute with common able to use a variety of
problems below
units of length. addition and subtraction
correctly.
strategies for mental
computation.

Step 1. The student should try to answer every question without a calculator and without help.
Step 2. Check the student’s answers using the solutions at the end of this document.
Step 3. The student should be given a second chance on problems answered incorrectly.

Solve each problem below using a ruler.

1. Draw two dots on the line below that are each 4 centimeters from dot D.

2. Draw a line below that is 4 inches long. Then, find the 2. ________ cm
length of the line you drew to the nearest centimeter.

Solve each problem below.

3. Write 273 centimeters in meters and centimeters. 3. ______ m _______ cm

4. Snorff is 7 ft 3 in tall. Glorff is 3 ft 7 in tall. How 4. _______ ft _______ in


much taller is Snorff than Glorff?

© 2018, AoPS Incorporated.


Are you ready for Beast Academy 2D?
Fill the boxes to make each equation true.

5. 554−86+ =555 6. 221+ −220=444

Fill the blanks in the skip-counting patterns below.

7. , , 33 , 44 , 55 , , , , , .

8. , , , , , 180 , 210 , 240 , , .

Solve each problem below.

9. Compute 41+44−14+41−44+41+14. 9. ____________

10. Compute (71+73+75+77)−(61+63+65+67). 10. ____________

11. Winnie’s number is half of 220. What is half of 11. ____________


Winnie’s number?

Fill every empty hexagon with an 8 or a 9 so the sum of the numbers in


every horizontal ( ) and diagonal ( or ) row of hexagons is odd.

12. 13.
9 8 8 8
8 9 8 8 8
8 9 8
8 9
8 9

© 2018, AoPS Incorporated.


Are you ready for Beast Academy 2D?
Solutions
1. We place the 0 cm mark of the ruler at the dot marked D 5. 555 is 1 more than 554. So, we add 1 more than we
and turn our ruler until the 4 cm mark lands on the line. subtract. Since we subtract 86, the number in the box is
We can do this on the right and on the left. We draw dots 87.
on the line at the 4 cm mark on both sides of the D.
554−86+ 87 =555.
6. Adding then taking away 220 gives the same result
7
5
6 as taking away 220 then adding .
D 3
4
Subtracting 220 from 221 gives 1. So, we must add 443
2
0 cm
1 to get 444.
So, the number in the box is 443.
221+ 443 −220=444.

7
7. We skip-count by 11’s to complete the pattern to the right
6
5 of 55.
4 +11 +11 +11 +11 +11 +11 +11
3
2
1
0 cm __, __, 33, 44, 55, 66, 77, 88, 99, 110.
Since the number to the right of 33 is 11 more than
D 33, the number to the left of 33 is 11 less than 33. We
2. First, we draw a line that is 4 inches long. continue this pattern to the left.
−11 −11
0 in 1 2 3 4 5 6
11, 22, 33, 44, 55, 66, 77, 88, 99, 110.

8. Since 240 is 30 more than 210, we are skip-counting by


30’s.
Then, we measure the line in centimeters. The line is
about 10 centimeters long. We skip-count by 30’s to complete the pattern to the right.
+30 +30 +30 +30

0 cm 1 2 3 4 5 6 7 8 9 10 11 12 13 14 15 , , , , , 180, 210, 240, 270, 300.

Since the number to the right of 180 is 30 more than 180,


the number to the left of 180 is 30 less than 180. We
3. 273 centimeters is more than 200 centimeters continue this pattern to the left.
(2 meters) but less than 300 centimeters (3 meters). −30 −30 −30 −30 −30
Since 273−200=73, we know 273 centimeters is 73
centimeters more than 2 meters. 30, 60, 90, 120, 150, 180, 210, 240, 270, 300.
So, 273 centimeters is 2 m 73 cm. 9. We cross out addition and subtraction that cancel, and
4. Snorff is 7 ft 3 in − 3 ft 7 in taller than Glorff. compute the expression as shown.

Since we can’t take 7 inches from 3 inches, we rewrite 41+44−14+41−44+41+14


7 ft 3 in as 6 ft 15 in. Then, we subtract. =41+44−14+41−44+41+14
+4
+444
4 −4
−4
44
4
6 ft 15 in =41 −14+41
−1
14
4 +41+14
+1
+1
14
4
7 fft 3 in
i =41 +41 +41
− 3 ft 7 in =123.
3 ft 8 in 10. 71 is 10 more than 61.
So, Snorff is 3 ft 8 in taller. 73 is 10 more than 63.
75 is 10 more than 65.
or 77 is 10 more than 67.
To find the difference between 7 ft 3 in and 3 ft 7 in, we So, (71+73+75+77) is 10+10+10+10=40 more than
can count up. (61+63+65+67).
We count up by 5 in to get from 3 ft 7 in to 4 ft. So (71+73+75+77)−(61+63+65+67)=40.
We count up by 3 ft 3 in to get from 4 ft to 7 ft 3 in.
11. Since 110+110=220, half of 220 is 110. So, Winnie’s
All together, we count up by 5 in + 3 ft 3 in = 3 ft 8 in. number is 110.
So, Snorff is 3 ft 8 in taller.
110 is 100+10. Half of 100 is 50, and half of 10 is 5.
So, half of Winnie’s number 110 is 50+5=55.
© 2018, AoPS Incorporated.
Are you ready for Beast Academy 2D?

12. We can tell whether a sum is even or odd by how many 13. Each row must have an odd number of 9’s. We complete
odd numbers are in the sum. If there is an even number the puzzle as shown below.
of odds, the sum will be even. If there is an odd number
Step 1: Step 2:
of odds, the sum will be odd.
Since 9 is odd and 8 is even, there must be an odd 8 8 9 8 8 9
number of 9’s in each row to get an odd sum. It does not
matter how many 8’s are in each row. 8 8 8 8 8 9 8
We fill the blanks so that each row has an odd number of 9 8 9 8 8
9’s as shown below. 9 9

Step 1: Step 2:
Step 3: Step 4:
9 8 8 9 8 8
8 9 8 8 9 8 8 8 9 8 8 9
8 9 8 9 8 8 9 8 8 8 9 8
8 8 9 8 9 8 9 9 8 9 8
8 8 9 8 8 9 9 9 9
8 8
Step 3: Step 4:
Step 5: Final:
9 8 8 9 8 8
8 8 9 8 8 8 9 8 8 8 9 8 8 9

8 9 9 8 8 9 9 8 8 9 8 8 8 9 8

9 8 9 8 9 9 9 8 9 8 9 9 8 9 8

8 8 9 8 8 9 8 9 9 8 9 9 9
9 8 8 9 8
Final:
You may have used a different set of steps to arrive at the
9 8 8 same answer.
8 8 9 8
8 8 9 9 9
9 9 8 9
8 8 9

You may have used a different set of steps to arrive at the


same answer.

© 2018, AoPS Incorporated.


Are you ready for Beast Academy 3A?

Before beginning
Beast Academy 3A,
a student should be
comfortable adding and
subtracting multi-digit
numbers and must have A student ready
a solid understanding for Beast Academy
of place value. 3A should be able to
answer at least 12 of
the 16 problems below
correctly.

Step 1. The student should try to answer every question without a calculator and without help.
Step 2. Check the student’s answers using the solutions at the end of this document.
Step 3. The student should be given a second chance on problems that he or she answered incorrectly.

Fill in each blank below with the number


that makes each equation true:

1. 3+41+47+9=________ 1. _________

2. 32+16+8+4+2+1+1=________ 2. _________

3. 555+666=________ 3. _________

4. 524−325=________ 4. _________

5. _____−12=75 5. _________

6. 200+ _____=567 6. _________

7. 800−82−17=800− _____ 7. _________

8. 97+397=500− _____ 8. _________

Write the following numbers with digits.

9. Five thousand, four hundred seventy-three. 9. _________

10. Sixteen thousand, fifty-two. 10. _________

© 2012, AoPS Inc.


Are you ready for Beast Academy 3A?

Answer each:

11. What is the sum of the thousands digit and the tens digit 11. _________
in 56,987?

12. Which number below has a ones digit that is greater than 12. _________
its ten-thousands digit?

90,287 26,532 41,902 47,658

13. Dave has the same number of action figures as Richard. 13. _________
Which of the following could be the number of action figures
Dave and Richard have all together?

15 23 46 79

14. Chelsea played 5 basketball games. In each game, she 14. _________
scored an odd number of points. Which of the following could
be the total number of points Chelsea scored in all 5 games?

10 22 33 40

15. What digit could fill both blanks below to make this five-digit 15. _________
number greater than 74,500 but less than 75,600?

7__,4__9

16. Akil was born 4 years before Kate. Ada is a year younger 16. _________
than Akil. If Ada is 9 years old, how many years old is Kate?

© 2012, AoPS Inc.


Are you ready for Beast Academy 3A?
Solutions
1. 3+41+47+9=(3+47)+(41+9)=50+50=100. 97 is very close to 100, and 397 is very close to 400,
2. 32+16+8+4+2+1+1= so the sum of 97+397 is very close to 100+400=500.
(32+8)+(16+4)+(2+1+1)=40+20+4=64. 97 is 3 less than 100, and 397 is 3 less than 400.
If we add 97 to 397, we get a number that is 3+3=6
or
less than 100+400=500.
If we reverse the order of the numbers we are adding So, 97+397=500−6.
(1+1+2+4+8+16+32), we are doubling each time we
9. “Five thousand, four hundred seventy-three” is five
add a number: thousands, four hundreds, seven tens and three ones:
1+1+2+4+8+16+32 5,473.
=2+2+4+8+16+32 10. “Sixteen thousand, fifty-two” is one ten-thousand, six
= 4+4+8+16+32 thousands, five tens, and two ones: 16,052.

= 8+8+16+32 11. The thousands digit in 56,987 is 6. The tens digit in


56,987 is 8. So, the sum of the thousands digit and the
= 16+16+32 tens digit is 6+8=14.
= 32+32 12. In each number, the ones digit is farthest to the right,
= 64. and the ten-thousands digit is farthest to the left. The
only number whose ones digit is greater than its ten-
3. 555+666=(500+50+5)+(600+60+6) thousands digit is 47,658.
=(500+600)+(50+60)+(5+6)
=1,100+110+11=1,221. 13. The sum of any number and itself is always even. For
example, 2+2=4 and 3+3=6. Richard and Dave have
or the same number of action figures, so, all together,
We could use the traditional addition algorithm: they must have an even number of action figures. 46
11 is the only even number listed, so Richard and Dave
555 could have 46 action figures all together. If Richard and
+666 Dave each have 23 action figures, they will have 46 all
together.
1,221
4. We are asked to find the difference, so we want to know
14. When we add two odd numbers, the result is always
even. When we add an even number and an odd
how much bigger 524 is than 325. If we add 325+200, we
number, the result is always odd. So,
get 525. So, 325+200−1=524.
after 2 games, the total number of points Chelsea
This means that 524 is greater than 325 by 200− 1=199,
scored was even (odd+odd=even).
so 524−325=199.
After 3 games, the total number of points Chelsea
or scored was odd (even+odd=odd).
We could use the traditional subtraction algorithm: After 4 games, the total number of points Chelsea
1 14 4 11 14 scored was even (odd+odd=even).
524 524 After 5 games, the total number of points Chelsea
−325 −325 scored was odd (even+odd=odd).
9 199 33 is the only odd answer choice.
5. Since we subtract 12 from some number to get 75, our 15. Since the number is less than 75,600, the thousands
answer is the number that is 12 more than 75. digit of 7_,4_9 must be 5 or smaller.
75+12=87, so 87−12=75. If we place a 5 in both blanks, 75,459 is greater than
6. Since we add some number to 200 to get 567, that 74,500 but less than 75,600.
number must be 200 less than 567. If we place a 1, 2, 3, or 4 in both blanks, we get a
567−200=367, so 200+367=567. number that is less than 74,500, so 5 is the only
possible choice.
7. When we subtract 82 then subtract 17, we subtract a total
of 82+17=99. So, 800−82−17=800−99.
16. We start with the one age we are given: Ada is 9 years
old. We are also told Ada is a year younger than Akil,
8. 97+397=494. So, we can write the question as so Akil is a year older than Ada: 9+1=10 years old.
494=500−___. Since 494 is 6 less than 500, Akil was born 4 years before Kate, so Akil is 4 years
494=500−6. older than Kate. We subtract 4 years from Akil’s age to
or get Kate’s age. This makes Kate 10−4=6 years old.

© 2012, AoPS Inc.


Are you ready for Beast Academy 3B?

Before beginning
Beast Academy 3B, a
student should be able
to classify polygons, skip-
count, and understand the A student ready
concepts of perimeter for Beast Academy
and area. 3B should be able to
answer at least 9 of the
13 problems below
correctly.

Step 1. The student should try to answer every question without a calculator and without help.
Step 2. Check the student’s answers using the solutions at the end of this document.
Step 3. The student should be given a second chance on problems that he or she answered incorrectly.

1. Connect 3 of the points below to 2. Connect 4 of the points below to


make an obtuse triangle. make a square.

3. Circle the shape or shapes below that cannot be made with seven dominoes ( ).

Fill in the blanks to complete each skip-counting pattern below:

4. 5, 10, 15, _____, 25, _____, _____, 40, ...

5. 13, 19, _____, _____, _____, 43, 49, _____, 61, ...

6. _____, _____, 23, _____, 31, _____, _____, 43, ...

© 2012, AoPS Inc.


Are you ready for Beast Academy 3B?
Answer each:

7. Ella begins at 9 and skip-counts by 7’s. Jack begins at 9 and skip- 7. ________
counts by 8’s. What is the next number that both Ella and Jack will
say?

8. What is the perimeter of a regular pentagon with sides of length 7? 8. ________

9. The height of a rectangle is increased by 3 inches. The new 9. ________


rectangle has a perimeter of 30 inches. What was the perimeter of
the original rectangle?

10. What is the perimeter of the rectilinear shape below? 10. ________
9

11. What is the area of the rectilinear shape below? 11. ________
9

6
10

12. Six squares are arranged as shown. The perimeter of each 12. ________
square is 5. What is the perimeter of the shape they make?

13. One side of a triangle is 3 inches long. Another side of the same
triangle is 6 inches long. Circle every value below that could be the
length of the triangle’s third side.

2 inches 5 inches 8 inches 11 inches 14 inches

© 2012, AoPS Inc.


Are you ready for Beast Academy 3B?
Solutions
1. An obtuse triangle must have one obtuse angle. 7. We write the first few numbers Ella says:
There is only one way to connect three of the points to 9, 16, 23, 30, 37, 44, 51, 58, 65, 72, 79, ...
make an obtuse triangle, as shown below. We also write the first few numbers that Jack says:
9, 17, 25, 33, 41, 49, 57, 65, 73, 81, ...
The next number that Ella and Jack both say is 65.
2. There is only one way to connect four of the points to 8. A regular pentagon has 5 sides with equal length, so the
make a square, as shown below: perimeter of a regular pentagon with sides of length 7 is
7+7+7+7+7=35.
9. When we make a rectangle 3 inches taller, the length of
each vertical side increases by 3.

3. If we color each shape with light and dark squares like a +3 +3


chess board, the first and fourth shapes each have 8 dark
squares and 6 light squares. The second and third shapes
have 7 light and 7 dark squares.
So, the perimeter of the original rectangle is 6 inches
less than the perimeter of the new rectangle. The new
rectangle has a perimeter of 30 inches, so the perimeter
of the original rectangle is 30−6=24 inches.
10. The short sides on the right side of the figure must add up
Since 7 dominoes will always cover 7 light squares to 5, and the short sides on the bottom of the figure must
and 7 dark squares, the first and fourth shapes are add up to 9, so the perimeter of the shape is the same as
impossible to make with dominoes. the perimeter of a 5 by 9 rectangle:
9

(9+5)+(9+5)=14+14=28.
Below are possible ways to make the other two shapes.
11. We can split the rectilinear shape into two rectangles as
follows: 9
9

6 6
4. We skip-count by 5’s, beginning at 5. 10
+5 +5 +5 +5 +5 +5 +5
4 4
5, 10, 15, 20, 25, 30, 35, 40, ... 3 3
5. We skip-count by 6’s, beginning at 13.
The area of the 6 by 9 rectangle is 9+9+9+9+9+9=54
+6 +6 +6 +6 +6 +6 +6 +6
squares.
13, 19, 25, 31, 37, 43, 49, 55, 61, ... The area of the 4 by 3 rectangle is 4+4+4=12 squares.
6. We look at the two closest numbers in the pattern: 23 and The area of the original shape is
31. We look for the number we can add to 23 twice to get 54+12=66 (squares).
31. Because 31−23=8, we need to add 8 to get from 23 12. Each square has 4 sides, and it takes 12 of those sides
to 31. Since 4+4=8, we can add 4 twice to get 8. So, we to make the perimeter of the new shape. 4+4+4=12, so
are skip-counting by 4’s. this is the same as the number of sides in three squares.
+4 +4 +4 +4 +4 Since the perimeter of each square is 5, the perimeter of
__, __, 23, 27, 31, 35, 39, 43, ... the new shape is 5+5+5=15.
When skip-counting by 4’s, we add 4 to get the next 13. The sum of the two shortest sides of a triangle must be
number. To get the previous number, we can do the greater than the length of the longest side. If 6 is the
opposite: subtract 4. We work backwards from 23 by longest side of the triangle, then the third side must be
subtracting to find the first two numbers in the pattern. longer than 6−3=3 inches. If the third side of the triangle
−4 −4 +4 +4 +4 +4 +4 is the longest, it must be shorter than 3+6=9 inches.
This leaves two possible lengths in the list:
15, 19, 23, 27, 31, 35, 39, 43, ...
2 inches 5 inches 8 inches 11 inches 14 inches

© 2012, AoPS Inc.


Are you ready for Beast Academy 3C?

Before beginning
Beast Academy 3C,
a student should have
a strong foundation in
basic multiplication,
perfect squares, and the A student ready
distributive property. for Beast Academy
3C should be able to
answer at least 10 of
the 14 problems below
correctly.

Step 1. The student should try to answer every question without a calculator and without help.
Step 2. Check the student’s answers using the solutions at the end of this document.
Step 3. The student should be given a second chance on problems that he or she answered incorrectly.

1. Complete the missing entries in the 2. Complete the missing entries in the
times table below: times table below:

× 8 4 3 × 5 8

49 70

72 3 18 60

8 48

6 42 200

3. Find the area of the rectangle below, in square units. 3. ________


43

4. What is the greatest possible area of a rectangle that has the same 4. ________
perimeter as the rectangle above (in square units)?

© 2012, AoPS Incorporated.


Are you ready for Beast Academy 3C?

Compute:

5. 42×5 5. ________

6. 8×251 6. ________

7. 17×7+23×7 7. ________

8. 5×9×2×2×4×5×5×3 8. ________

9. (115×115)−(114×114) 9. ________

10. 9×79×9 10. ________

11. What is the sum of the first thirty odd numbers: 11. ________
1+3+5+7+ · · · +55+57+59?

Answer each:

12. Rae, Sara, and Taj each have a perfect square number of 12. ________
books. All together, they have 93 books. Taj has the most
books. How many books does Taj have?

13. Gumballs cost 6 cents each. Chocolates cost 11 cents each. 13. ________
What is the cost in cents of 19 gumballs and 6 chocolates?

14. Captain Kraken has 7 treasure chests. Each chest holds 95 14. ________
rubies. He gives 45 rubies to each of his 7 crew members.
How many rubies does Captain Kraken have left?

© 2012, AoPS Incorporated.


Are you ready for Beast Academy 3C?
Solutions
1. The missing entries are filled in below in bold. 6. We use the distributive property:
× 8 4 7 3 251×8=(200+50+1)×8
7 56 28 49 21 =(200×8)+(50×8)+(1×8)
=1,600+400+8
9 72 36 63 27
=2,008.
2 16 8 14 6 or
6 48 24 42 18 Eight is equal to 2×2×2. To multiply a number by 8, we
2. The missing entries are filled in below in bold. can double the number three times:
251×8=(251×2)×2×2
× 6 5 20 8
=(502×2)×2
70 420 350 1400 560 =1,004×2
3 18 15 60 24 =2,008.
6 36 30 120 48 7. We use the distributive property:
17×7+23×7=(17+23)×7
40 240 200 800 320
=40×7
3. To find the area of a rectangle, we multiply its height by =280.
its width. To find the area of a 7-by-43 rectangle, we can 8. Since multiplication is commutative and associative, we
split it into two smaller rectangles whose areas are easy can multiply the numbers in any order. So, we can pair
to compute. We add the areas of the smaller rectangles to numbers whose products are multiples of ten:
find the area of the larger rectangle:
40 3 5×9×2×2×4×5×5×3=(5×2)×(2×5)×(4×5) ×(9×3)
=(10×10)×(20×27)
7 7
=100×540
7×43=7×40+7×3=280+21=301. =54,000.
The area of the rectangle is 301 (square units).
4. The perimeter of the rectangle is 43+7+43+7=100.
9. In Beast Academy 3B, we learn that we don’t need to
calculate these squares to calculate their differences!
For a given perimeter, a square is the rectilinear shape
115×115=(114×114)+(114+115), so
with the greatest area. Since 25 ×4=100, a square with
115×115=114×114+229.
perimeter 100 has side length 25. The area of a square
115×115 is 229 more than 114×114, so we have
with side length 25 is 25×25 (square units).
(115×115)−(114×114)=229.
We can use the distributive property to compute 25×25.
25×25=25×(20+5)=25×20+25×5=500+125 10. First, we multiply 9×79.
Since 79=80−1, we have
=625 (square units).
9×79=9×(80−1)
or =(9×80)−(9×1)
In Beast Academy 3B, we learn another way to square =720−9=711.
any number that ends in 5. Here’s how it works with 25: Then, we multiply 711 by 9.
25×25=(20×30)+25=600+25=625 (square units). 711×9=(700+10+1)×9
=(700×9)+(10×9)+(1×9)
5. We can use the distributive property:
=6,300+90+9=6,399.
42×5=(40+2)×5
=(40×5)+(2×5) or
=200+10 We can multiply the 9’s first:
=210. 9×79×9=(9×9)×79=81×79. In Beast Academy 3B,
or we learned an easy way to compute the product of
two numbers that differ by 2. We find the square of the
Five is half of 10. To multiply a number by 5, we can
number between them, then subtract 1.
multiply the number by 10, then find half the result.
So, 81×79=(80×80)−1=6,400−1=6,399.
42×10=420. Half of 420 is 210.

© 2012, AoPS Incorporated.


Are you ready for Beast Academy 3C?
11. The sum of the first thirty odd numbers is equal to 30
squared. 30×30=900, so the sum of the first thirty odd
numbers is 900.
or
Since addition is commutative and associative, we can
add the numbers in any order we want. So, we rearrange
these numbers, pairing numbers whose sum is 60:
1+3+5 · · · +57+59=(1+59)+(3+57)+ · · ·+(29+31)
All together, there are 15 pairs of numbers whose sum is
60. So, the sum is equal to 15×60.
We use the distributive property to evaluate 15×60:
15×60=(10+5)×60=10×60+5×60=600+300=900.
12. Since each person has a perfect square number of books,
we look for a way to write 93 as the sum of three perfect
squares. The perfect squares less than 93 are 1, 4, 9, 16,
25, 36, 49, 64, and 81. The only way to write 93 as the
sum of three perfect squares is 93=64+25+4.
Since Taj has the most books, he has 64 books.
13. Since gumballs cost 6 cents each, 19 gumballs cost 19×6
cents. Chocolates cost 11 cents each, so 6 chocolates
cost 6×11 cents. The total cost in cents of gumballs and
chocolates is (19×6)+(6×11). We can use the distributive
property to factor and compute (19×6)+(6×11):
(19×6)+(6×11)=(19×6)+(11×6)
=(19+11)×6
=30×6
=180.
So, 19 gumballs and 6 chocolates cost 180 cents.
14. All together, Captain Kraken has 7×95 rubies, and
he gives away 7×45 rubies. Then, he is left with
7×95−7×45 rubies.
To calculate 7×95−7×45, we can use the distributive
property: 7×95−7×45=7×(95−45)=7×50.
7×50=350, so Captain Kraken is left with 350 rubies.

© 2012, AoPS Incorporated.


Are you ready for Beast Academy 3D?

Before beginning
Beast Academy 3D, a
student should have a basic
understanding of variables, be
able to compute quotients with A student
remainders, and be familiar ready for Beast
with most common units of Academy 3D should
measurement. be able to answer at
least 11 of the 16
problems below
correctly.

Step 1. The student should try to answer every question without a calculator and without help.
Step 2. Check the student’s answers using the solutions at the end of this document.
Step 3. The student should be given a second chance on problems that he or she answered incorrectly.

Solve for the variable in each equation below:

1. 54=9×w 2. 37+n=115 1. w= _____

2. n =_____

3. Evaluate w×10+6 for w=45. 3. ________

4. Evaluate 300−2×k for k=10. 4. ________

5. Simplify 9+y−4+y+12−y. 5. ____________

6. Write an equation with the same meaning as the sentence below. 6. ____________
Then, solve for m.

Seven less than m is sixty-six. m=_____

7. When 35 is divided by 8, the quotient is a with remainder b. What 7. ________


number can be divided by 8 to get quotient b with remainder a?

8. What is the side length in inches of a regular hexagon that has a 8. ________
perimeter of 8 feet?

9. What is the remainder when 24×17 is divided by 7? 9. ________

© 2013, AoPS Incorporated.


Are you ready for Beast Academy 3D?

10. Grogg arranges toothpicks to make hexagons as shown below. It takes 10. ________
11 toothpicks to make 2 hexagons, 16 toothpicks to make 3 hexagons,
and 21 toothpicks to make four hexagons. How many toothpicks will
Grogg need to make 50 hexagons if he continues this pattern?

···

11. What will the time be 93 minutes after the time shown on the clock 11. ________
below?
11 12 1
10 2
9 3
8 4
7 5
6

12. Fill in the empty white squares in the puzzle below so that each of the
five equations in the puzzle is true.

54 ÷ 3 =
× − ÷
30 ÷ =

= =
45 ÷ 15 =

13. How many seconds are in two hours? 13. ________

14. How many inches are in five yards? 14. ________

15. The perimeter of a square is one meter. What is the length 15. ________
in centimeters of one side of the square?

16. Dara has nine U. S. coins for a total of 68 cents. 16. ________
How many nickels does she have?

© 2013, AoPS Incorporated.


Are you ready for Beast Academy 3D?
Solutions

1. 9× 6 =54, so w=6. 8. Since 1 foot is equal to 12 inches, 8 feet equals


8×12=96 inches.
2. To solve the equation, we subtract 37 from both sides:
37+n=115 A regular hexagon has 6 sides of equal length. So, to find
−37 −37 the side length of the hexagon, we divide 96 by 6:
n = 78 10+6
So, n=78. 6 96
−60
3. When w=45, the expression w×10+6 is equal to
36
45×10+6=450+6=456. −36
4. When k=10, the expression 300−2×k is equal to 0
300−2×10=300−20=280.
Since the remainder is 0, we can write 96÷6=10+6=16.
5. We use the associative and commutative properties of
So, the side length of a regular hexagon with a perimeter of
addition to rewrite the expression:
8 feet is 96÷6=16 inches (in).
9+y−4+y+12−y=(9+y)−4+(y+12)−y
9. We first multiply 24×17, then divide by 7 to find the
=(y+9)−4+(12+y)−y
remainder.
=y+(9−4+12)+y−y
=y+17+y−y 24×17=24×(20−3)=480−72=408.

Then, starting with y+17, adding y and subtracting y is the So, 24×17=408. Next, we divide 408 by 7:
same as doing nothing. So, y+17+y−y=y+17. 50+8
9+y−4+y+12−y simplifies to y+17 (or 17+y). 7 408
−350
6. “Seven less than m” means m−7 (not 7−m), and “is” 58
means “equals.” Therefore, our equation is m−7=66. −56
To solve the equation, we add 7 to both sides: 2

m−7=66 So, 24×17 has remainder 2 when divided by 7.


+7 +7 or
m = 73 We begin by finding the remainder when each number is
divided by 7.
So, m=73.
24÷7 has remainder 3.
7. First, we divide 35 by 8: 17÷7 has remainder 3.
4
Then, we multiply the remainders.
8 35 (24×17)÷7 has the same remainder as (3×3)÷7.
−32
3×3=9, and 9÷7 has remainder 2.
3
So, (24×17)÷7 has remainder 2.
35÷8 has quotient 4 and remainder 3. So, a=4 and b=3. 10. To make the first hexagon, Grogg needs 6 toothpicks.
We are looking for a number that has quotient 3 and To make each additional hexagon, he must add 5 more
remainder 4 when divided by 8. This means that 8 goes
toothpicks. So to make 50 hexagons, Grogg needs 6×1
into our number 3 times, with 4 left over.
toothpicks for the first and 5×49 toothpicks for the next 49.
8×3 is 24, plus 4 extra is 28. All together, he needs 6+(5×49)=6+245=251 toothpicks.
So, 28 has quotient 3 and remainder 4 when divided by 8. or
We could think of Grogg starting with 1 toothpick and
adding 5 toothpicks for each hexagon. So, to make n
hexagons, Grogg starts with 1 toothpick and adds 5×n
more toothpicks for a total of 1+5×n toothpicks.
To make 50 hexagons, Grogg needs
1+(5×50)=1+250=251 toothpicks.

© 2013, AoPS Incorporated.


Are you ready for Beast Academy 3D?
11. The time shown on the clock is 6:47. 13. Since there are 60 seconds in 1 minute and 60 minutes in
13 minutes after 6:47 is 7:00. 1 hour, one hour equals 60×60=3,600 seconds.
This leaves 93−13=80 minutes to add. So, two hours equal 3,600×2=7,200 seconds.
60 minutes (1 hour) after 7:00 is 8:00.
This leaves 80−60=20 minutes to add. 14. Since there are 3 feet in 1 yard, and 12 inches in 1 foot,
20 minutes after 8:00 is 8:20. one yard equals 3×12=36 inches.
So, 93 minutes after 6:47 is 8:20. So, 5 yards equal 5×36=180 inches (in).
12. We start with the horizontal equation at the top: 54÷3. 15. One meter equals 100 centimeters, so the perimeter of the
We use long division to compute 54÷3=18, so we have square is 100 centimeters. A square has 4 sides of equal
length, so we calculate the side length by dividing the
54 ÷ 3 = 18 perimeter by 4:
× − ÷
100÷4=25.
30 ÷ =
The side length of a square with a perimeter of 1 meter is
= =
25 centimeters (cm).
45 ÷ 15 =
16. Using nickels, dimes, and quarters, we can only get a
Then, we solve the center vertical equation: 3× =15. number of cents that is a multiple of 5.
Since 3× 5 =15, we have Dara has 9 coins worth 68 cents. Since 68 is not a
54 ÷ 3 = 18 multiple of 5, we know that Dara must have at least 3
× − ÷
pennies. That leaves 9−3=6 coins worth 68−3=65¢.
30 ÷ 5 = If all 6 of the remaining coins were only dimes, nickels or
= = pennies, then the coins would be worth 10×6=60 cents
or less. That’s not enough!
45 ÷ 15 =
So, Dara must have at least one quarter. That leaves
Next, we solve the center horizontal equation: 30÷5. 6−1=5 coins worth 65−25=40¢.
Since 30÷5=6, we have If Dara had a second quarter, that would leave 4 coins to
54 ÷ 3 = 18 make 40−25=15¢. We cannot make 15¢ with 4 coins, so
× − ÷ Dara only has one quarter.
30 ÷ 5 = 6 If all 5 remaining coins were nickels, they would be worth
= = only 5×5=25¢. So, we must have at least 1 dime.
45 ÷ 15 = We look for a way to make 40¢ with 5 coins, all of which
are dimes and nickels.
Then, we can use either the right vertical equation or the
bottom horzontal equation to find the correct number to Dimes Nickels Total Value
place in the final square. 1 4 10+20=30¢
18÷6=3 and 45÷15=3 2 3 20+15=35¢
So, we have 3 2 30+10=40¢
4 1 40+5=45¢
54 ÷ 3 = 18
5 0 50¢
× − ÷
30 ÷ 5 = 6
= = We can only make 40¢ with 3 dimes and 2 nickels.
45 ÷ 15 = 3 So, all together, Dara has 1 quarter, 3 dimes, 2 nickels,
and 3 pennies for a total of 68¢.

© 2013, AoPS Incorporated.


Are you ready for Beast Academy 4A?

Before beginning
Beast Academy 4A, a
student should understand
the basics of perimeter A student ready
and area, multiplication and for Beast Academy
division, measurement, 4A should be able to
fractions, and answer at least 10 of
estimation. the 14 problems below
correctly.

Step 1. The student should try to answer every question without a calculator and without help.
Step 2. Check the student’s answers using the solutions at the end of this document.
Step 3. The student should be given a second chance on problems that he or she answered incorrectly.

Compute.

1. 27×6=________ 2. 199×80=________

Compute the quotient and remainder for each division expression.

3. 65÷11 3. Quotient: _____ Remainder: _____

4. 196÷9 4. Quotient: _____ Remainder: _____

5. A regular hexagon has a perimeter of 108 inches 5. n =________


and side length n inches. What is the value of n?

6. Find the perimeter and area of the rectilinear figure below. 6. Perimeter: ________
12 ft Area: ________

4 ft 4 ft

2 ft 5 ft
3 ft

7. How many square feet are there in 2 square yards? 7. ________

8. Compute (85×85)−(84×84). 8. ________


© 2013, AoPS Incorporated.
Are you ready for Beast Academy 4A?

9. Place each of the four numbers below in one of


the empty circles in the diagram on the right so 24
that no two connected circles have a sum that
has remainder 0 when divided by 5.
22

Missing Numbers: 26, 27, 28, 29

10. Connect each expression on the left to an equal 13×57 2,001


value on the right by estimating the value of
each product. 23×87 1,331

11×121 741

11. Label each value on the number line below with a mixed number in simplest form.

4 5 6

63
7

25
12. The fraction 9 is closest to which whole number? 12. ________

13. Use the given fractions to label the three unlabled points on the number line below:

Fractions: 59 , 58 , 2
5 1
0 2 1

14. Four congruent right triangles 5 cm 2 cm 14. ______ square cm


are arranged as shown to form 2 cm
a large square, with a smaller
tilted square within. What is the
area of the tilted square?
5 cm

© 2013, AoPS Incorporated.


Are you ready for Beast Academy 4A?
Solutions
1. 27×6=(20+7)×6=(20×6)+(7×6)=120+42=162. We split the shape into two rectangles.
2. 199×80=(200−1)×80 12 ft 12 ft
=(200×80)−(1×80)
4 ft 4 ft 4 ft
=16,000−80
=15,920. 2 ft 5 ft
3 ft 3 ft

3. 5 The quotient of 65÷11 is 5, and the 5 ft 5 ft

11 65 remainder is 10. The area of the 12 ft by 4 ft rectangle is 12×4=48 sq ft.


−55 The area of the 5 foot by 3 foot rectangle is
10
5×3=15 sq ft. The total area of the rectilinear shape is
48+15=63 square feet (sq ft).
4. 20+1 The quotient of 196÷9 is 20+1=21,
7. A square yard is 3 ft by 3 ft. So, there are
9 196 and the remainder is 7. 3×3=9 square feet in 1 square yard.
−180
In 2 square yards, there are 2×9=18 square feet (sq ft).
16
−9 8. We don’t need to compute (85×85) and (84×84) to find
7 their difference. We can get from one perfect square to
the next by adding. See Beast Academy 3B for a more
5. A regular hexagon has 6 sides of equal length. To find the thorough explanation.
length of each side, we divide its perimeter by 6.
To get from 84×84 to 85×84, we add 84.
10+8
To get from 85×84 to 85×85, we add 85.
1086 So, (84×84)+84+85=(85×85).
−60
48 This means that (85×85) is 84+85=169
−48 more than (84×84).
0 Therefore, (85×85)−(84×84)=169.
108÷6 has remainder 0, so 108÷6=10+8=18.
9. If a number ends in a 0 or a 5, then its remainder is 0
The side length of a regular hexagon with a perimeter of
when divided by 5. We cannot place two numbers whose
108 inches is 18 inches. So, n=18.
sum ends in 0 or 5 in connected circles.
6. The short sides on the left side of the rectilinear shape 28+22=50, so 28 cannot be connected to 22. This
must add up to the same length as the short sides on leaves only the bottom right circle for 28.
the right side of the shape (3+4=7 ft). The short sides
26+24=50, so 26 cannot be connected to 24. This
on the bottom of the rectilinear shape must add up to the
leaves only the top left circle for 26.
same length as the long side on top (12 ft). So, the shape
has the same perimeter as a 12 ft by 7 ft rectangle. The 26 24
perimeter of the shape is 12+7+12+7=38 feet.
12 ft
22 28

7 ft
Then, 27+28=55, so 27 cannot be connected to 28.
That leaves only the top middle circle for 27.
The 29 fills the remaining circle as shown.
To find the area of the shape, we begin by finding the
26 27 24
length of the missing short side on the bottom of the
shape. The width of the rectilinear shape is 12 feet.
2+ 5 +5=12, so the missing horizontal length is 5 ft. 22 29 28
12 ft

4 ft 4 ft

2 ft 5 ft
3 ft

5 ft

© 2013, AoPS Incorporated.


Are you ready for Beast Academy 4A?
10. We estimate the products on the left: 13. Only one marked point on the number line is less than 2 .
1
13×57≈10×60=600, 2
23×87≈20×90=1,800, and Since 5
is the only given fraction whose numerator is less
2
11×121≈10×120=1,200. than half its denominator, 5 is the only fraction that is less
1
Then, we compare these estimates to the values on the than 2 :
right. Since 600 is closest to 741, we connect 13×57 to
741. Since 1,800 is closest to 2,001, we connect 23×87
2 1
to 2,001. Since 1,200 is closest to 1,331, we connect 0 5 2 1
11×121 to 1,331. 5 5
Eighths are larger than ninths, so > .
8 9
We label the
13×57 2,001 points as shown:

23×87 1,331
2 1 5 5
0 5 2 9 8 1
11×121 741
5 5
You may have also converted 8 and 9 to equivalent
11. The tick marks split the number line into 7 equal pieces fractions with the same denominator to compare them:
between each whole number. So, the tick marks identify 5 45
=
5 40 45 40
and 9 = 72 . Since 72 > 72 , we have 8 > 9 .
5 5
sevenths on the number line. 8 72

The far-right arrow marks six sevenths more than 6, so 14. The side length of the large square formed by attaching
6 the triangles is 5+2=7 centimeters.
we label it 6 7 . The middle-right arrow marks two sevenths
2
more than 5, so we label it 5 7 . The middle-left arrow We calculate the area of the tilted square by subtracting
5 the areas of the 4 congruent right triangles from the area
marks five sevenths more than 4, so we label it 4 7 .
28 of the larger square they create.
The far-left arrow marks three sevenths less than 4= 7 ,
25
so the left arrow marks 7 . Since 3 =3, 7 is four
21 25 The area of the larger square is 7×7=49 sq cm.
4 This is the shaded area below:
sevenths more than 3. We label the far-left arrow 3 7 .
5 cm 2 cm
2 cm
4 5 6
4 5 2 3 6
37 47 57 67 67 7 cm
5 cm

25 18 27
12. 9
is between 9 =2 and 9 =3. 7 cm
25 27 18 The area of each congruent triangle is
9
is two ninths from 9
and seven ninths from 9
.
2×5÷2=10÷2=5 sq cm. The total area of the four
18 25 27 triangles is therefore 4×5=20 sq cm. Subtracting the
9 9 9 areas of the four triangles, we are left with the shaded
area below:
5 cm 2 cm
2 3
2 cm
25 27
So, 9 is closest to 9 =3.
7 cm
5 cm

7 cm
The area of the tilted square is 49−20=29 square cm.

© 2013, AoPS Incorporated.


Are you ready for Beast Academy 4B?

         

        

                

      

                

      

 

         

         

                    

 
           

             

           

       

 
      

   

 

Step 1. The student should try to answer every question without a calculator and without help.
Step 2. Check the student’s answers using the solutions at the end of this document.
Step 3. The student should be given a second chance on problems that he or she answered incorrectly.

1. Label point S on the dot grid 2. Label point R on the dot grid
below so that quadrilateral BEST below so that quadrilateral GRID
is a parallelogram. is a kite.

B E G

T I

3. The measure of angle AXB is twice the measure


B
of angle BXC. Without using a protractor, find the
measure of angle BXC.

A X C

4. Shade the square that could be removed 5. Draw a cut line along the grid to split
from the shape below to leave a shape the shape into two pieces so that both
with at least one line of symmetry. pieces have rotational symmetry.

© 2014, AoPS Incorporated.


Are you ready for Beast Academy 4B?

Compute each product.

6. 9×327=_______ 7. 27×64=_______

8. 17×2,003=_______ 9. 2,020×701=_______

Rewrite each multiplication expression as a power.

10. 2×2×2×2=_______ 11. 13×13×13×13×13 =_______

Compute the value of each power.

12. 82=_______ 13. 54=_______

Fill in the missing exponent in each equation.

14. 23×27=2 15. 7×7 =75

Answer each question below.

16. How can the number 80 be written as the sum of two 16. 80=_____+_____.
powers of 2?

17. The product 147×594×912 ends with the digit ______.

18. Write 493 as a power of 7. 18. __________

© 2014, AoPS Incorporated.


Are you ready for Beast Academy 4B?
Solutions
1. A parallelogram is a four-sided polygon with two pairs of We are given that the measure of angle AXB is twice
opposite sides that are parallel. We draw parallelogram the measure of angle BXC. So, if we let a represent the
BEST by connecting B to E to S to T to B. So, segments measure in degrees of angle BXC, then the measure of
BE and TB are sides of the parallelogram. angle AXB is (a+a).
B E
B
(a+a)
a
T A X C
Then, we draw a line parallel to segment BE that passes
Since the sum of the measures of angles AXB and BXC is
through point T, and a line parallel to segment BT that
passes through point E. We label the intersection of these 180 degrees, we have a+(a+a)=180.
two points “S” to make parallelogram BEST. We simplify the left side to rewrite the equation as
B E 3×a=180. Since 3× 60 =180, we have a=60.
So, the measure of angle BXC is 60.
4. If we remove the shaded square, then we are left with the
S
figure that has the line of symmetry shown below:
T

2. We draw kite GRID by connecting G to R to I to D to G.


So, segments ID and DG are sides of the kite.
G

D
I 5. If we make the cut shown, then we create two shapes,
each with rotational symmetry.
Kites have two pairs of adjacent sides that are congruent,
but segments ID and DG are not the same length. So,
segment GR is the same length as segment DG, and
segment RI is the same length as segment ID.
There are two other segments in the grid with endpoint G
that has the same length as segment DG:
G
Both shapes have rotational symmetry of order 2.

D
I

Only one of these endpoints will make a kite when its


other endpoint is connected to point I, so we label that
point R. G R

6. We set up the numbers as shown.


D
327
I
×9

Distributing the 9 gives three partial products: 9×7=63,


3. The sum of the measures of angles AXB and BXC equals 9×20=180, and 9×300=2,700.
the measure of straight angle AXC. So, the sum of the We stack the partial products so that the units, tens, and
measures of angles AXB and BXC is 180. hundreds digits line up, as shown below.
327 327 327
B ×9 ×9 ×9
63 63 63
180 180
A X C 2,700

© 2014, AoPS Incorporated.


Are you ready for Beast Academy 4B?
Then, we add the partial products: 9. We distribute as shown below. We do not need to write a
327 partial product of 0.
×9
2,020 2,020 2,020
63 × 701 × 701 × 701
180
2,020 2,020 2,020
+ 2,700
2,943
2,020 3,020 2,020
7. Each number represented by a digit in 27 is multiplied by
× 701 × 701 × 701
each number represented by a digit in 64.
2,020 2,020 2,020
Distributing gives four partial products, 4×7=28, 14,000 14,000 14,000
4×20=80, 60×7=420, and 60×20=1,200. 1,400,000
27 27 27 27
Finally, we add the partial products.
× 64 × 64 × 64 × 64
28 28 28 28 2,020
80 80 80 × 701
420 420 2,020
1,200 14,000
Then, we add the partial products. + 1,400,000
27
× 64 1,416,020
28 10. We multiply four 2’s, so 2 is the base and 4 is the
80 exponent: 2×2×2×2=24.
420 11. We multiply five 13’s, so 13 is the base and 5 is the
+ 1,200 exponent: 13×13×13×13×13=135.
1,728 12. We evaluate 82 by multiplying two 8’s:
8. First, we distribute the 7 in 17. This gives us four 82=8×8
partial products: 7×3=21, 7×0=0, 7×0=0 and =64.
7×2,000=14,000.
13. We evaluate 54 by multiplying four 5’s:
It is not necessary to write a partial product of 0, since
adding 0 does not change a sum. 54=5×5×5×5
2,003 2,003 2,003 2,003 =25×5×5
× 17 × 17 × 17 × 17 =125×5
21 21 21 21 =625.
14,000 14. We write the powers as products:
Next, we distribute the 1 that is the tens digit of 17. Since 23×27=(2×2×2)×(2×2×2×2×2×2×2)
we know that 10×2,003=20,030 we do not need to =2×2×2×2×2×2×2×2×2×2.
distribute the 10 to each digit in 2,003 separately.
All together, we multiply ten 2’s, which equals 2 to the 10th
2,003 power: 23×27=2 10 .
× 17
or
21
3 7
14,000 When we evaluate 2 ×2 , we multiply three 2’s and then
20,030 another seven 2’s. All together, we multiply 3+7=10 twos,
which is equal to 210. So, 23×27=2 10 .
Finally, we add the partial products.
2,003
15. The product 7×7 is equal to 75.
× 17 75 is the product of five 7’s. So, we need to multiply 7 by
four more 7’s to get 75. Therefore, 7×7 4 =75.
21
14,000 16. We look at the powers of 2 between 1 and 80:
+ 20,030 21=2, 22=4, 23=8, 24=16, 25=32, and 26=64.
34,051 16+64=80, and the sum of any other pair is smaller or
larger than 80. So, the only two powers of 2 that we can
add to get 80 are 24=16 and 26=64.
So, we write 80=24+26.

© 2014, AoPS Incorporated.


Are you ready for Beast Academy 4B?
Since addition is commutative, you may have instead
written 80=24+26.
17. The units digit of 147×594×912 is the same as the units
digit of 7×4×2=56.
So, the product 147×594×912 ends with the digit 6.
In fact, 147×594×912=79,634,016.
18. We know 493=49×49×49. Since 49=7×7, we replace
each 49 with 7×7 to get
493=49×49×49
=(7×7)×(7×7)×(7×7).
So, 493 is the product of six 7’s. Therefore, 493=76.

© 2014, AoPS Incorporated.


Are you ready for Beast Academy 4C?

Before beginning
Beast Academy
4C, a student should
understand some basic A student
counting strategies, long ready for Beast
division, divisibility rules, Academy 4C should
and the basics of be able to answer at
fractions. least 13 of the 19
problems below
correctly.

Step 1. The student should try to answer every question without a calculator and without help.
Step 2. Check the student’s answers using the solutions at the end of this document.
Step 3. The student should be given a second chance on problems that he or she answered incorrectly.

1.
How many even numbers are there from 6 to 54? 1. ________

6, 8, 10, ... , 50, 52, 54

2. How many three-digit numbers have three odd digits? 2. ________

3. How many different arrangements of the letters in the word MATH 3. ________
are possible, including M-A-T-H?

4. In a tennis tournament with 10 players, each player competes in 4. ________


exactly one match with every other player. How many matches are
played?

Compute each quotient.

5. 350,000÷500=_______ 6. 2,400,000÷8,000=_______

7. 453,618÷9=_______ 8. 12,362,400÷6=_______

9. 1,450÷5=_______ 10. 345÷15=_______

© 2014, AoPS Incorporated.


Are you ready for Beast Academy 4C?

First, estimate each quotient. Then, compute the quotient and remainder.

11. 198÷23 is closest to: (circle one) 12. 9,570÷47 is closest to: (circle one)
2 10 50 200 2 10 50 200

quotient=_____ quotient=_____
23 198 47 9,570
remainder=_____ remainder=_____

13. Circle the numbers below that are divisible by 4, box the numbers that are divisible by 25,
and underline the numbers that are divisible by 100.
187 275 300 360 535 600 625 708 1,000

14. Circle all of the numbers below that 2,375 is divisible by.

2 4 5 12 25 50 1,000

17
15. Label 4 on the number line below as a mixed number.
4 5 6

Place a <, >, or = to compare each pair of fractions below.

16. 3 4 17. 3 3 18. 5 10


7 7 11 13 17 37

19. Write each of the digits 1 through 8 exactly once in the grid below so that there
are two digits in each row and two digits in each column. A number to the
left of the grid gives the product of the two numbers in that row. Similarly, a
number above the grid gives the product of the two numbers in that column.
10 8 24
2

24
20

© 2014, AoPS Incorporated.


Are you ready for Beast Academy 4C?
Solutions
1. Each number in this list is a multiple of 2. To make the list Finally, once we have chosen the first three letters, there
easier to count, we divide each number in the list by 2. is only 1 remaining letter to be the fourth.
6, 8, 10, …, 50, 52, 54 All together, there are 4×3×2×1=24 ways to arrange the
÷2 ÷2 ÷2 … ÷2 ÷2 ÷2 four letters in some order. You may have also written this
3, 4, 5, …, 25, 26, 27 as 4! or “4 factorial.”

Subtracting 2 from each number in the list gives us a list 4. Each game is played by one pair of players. In choosing
from 1 to 25. a pair of players, we have 10 choices for the “first” player
and 9 choices for the “second” player.
Now, the numbers are counted for us! So, there are 25
even numbers from 6 to 54. Since “Winnie vs. Alex” is the same game as “Alex vs.
Winnie,” the order in which we pick the players does not
2. There are five odd digits: 1, 3, 5, 7, and 9. So, to make matter. So, 10×9=90 counts each game twice.
a three-digit number with three odd digits, we have 5
choices for the hundreds digit, 5 choices for the tens digit, Therefore, there are 90÷2=45 games in the tournament.
and 5 choices for the ones digit. 5. We can divide 3,500 hundreds into groups of 5 hundreds
All together, there are 5×5×5=5 =125 three-digit
3 to make 3,500÷5=700 groups of 5 hundreds.
numbers with three odd digits. So, 350,000÷500=3,500÷5=700.
3. We create a diagram to organize which letter comes first, 6. 2,400,000÷8,000=2,400÷8=300.
second, third, and fourth in the arrangement. 7. Consider dividing 453,618 marbles into 9 buckets.
1st 2nd 3rd 4th We begin by dividing 450,000 of the marbles into 9
M T 1. AHMT buckets, which gives us 450,000÷9=50,000 marbles in
H
T M 2. AHTM
each bucket.
H T 3. AMHT
A M
T H 4. AMTH Then, we divide 3,600 more marbles among the 9
H M 5. ATHM buckets, which gives us 3,600÷9=400 more marbles in
T
M H 6. ATMH each bucket.
M T 7. HAMT Finally, we divide the 18 remaining marbles among the 9
A
T M 8. HATM
buckets, which gives us 18÷9=2 more marbles in each
A T 9. HMAT
H M bucket.
T A 10. HMTA
A M 11. HTAM So, each bucket has a total of 50,000+400+2=50,402
T
M A 12. HTMA marbles.
H T 13. MAHT Therefore, 453,618÷9=50,402.
A
T H 14. MATH
A T 15. MHAT or
M H
T A 16. MHTA
453,618÷9=(450,000+3,600+18)÷9
A H 17. MTAH
T
18. MTHA
=(450,000÷9)+(3,600÷9)+(18÷9)
H A
=50,000+400+2
H M 19. TAHM
A
M H 20. TAMH =50,402.
A M 21. THAM 8. 12,362,400÷6=(12,000,000+360,000+2,400)÷6
T H
M A 22. THMA =(12,000,000÷6)+(360,000÷6)+(2,400÷6)
A H 23. TMAH =2,000,000+60,000+400
M
H A 24. TMHA
=2,060,400.
All together, there are 24 ways to arrange these letters.
9. In a division problem, we can double both the dividend
or and the divisor without changing the quotient.
We count the arrangements without listing them all. We 1,450÷5=(1,450×2)÷(5×2)=2,900÷10=290.
begin by choosing one letter to be the first. There are 4
Remember that fractions are another way to write division,
choices for the first letter (M, A, T, or H).
and we can use multiplication to write equivalent fractions:
Then, no matter which letter we chose as the first, there ×2
are always 3 choices for the second letter.
1,450 2,900
1,450÷5= = =2,900÷10=290.
Similarly, no matter which two letters we chose to be first 5 10
and second, there are always 2 choices remaining for the ×2
third letter.
10. 345÷15=(345×2)÷(15×2)=690÷30=23.

© 2014, AoPS Incorporated.


Are you ready for Beast Academy 4C?
You may have taken different steps to find the same
estimate, quotient, and remainder. 14. The units digit of 2,375 is not 0, 2, 4, 6, or 8, so 2,375 is
11. 198 is close to 200, and 23 is close to 20. So, we estimate not divisible by 2.
that 198÷23 is about 200÷20=10. 4=2×2, so any multiple of 4 is a multiple of 2 and
23×5=115. So, 23 can go into 198 at 5 therefore even. Since 2,375 is not even, 2,375 is not
23 198 divisible by 4.
least 5 times. We subtract 115 from 198
−115
and have 83 left over. Since 83 is more 83 2,375 ends in 5, so 2,375 is divisible by 5.
than 23, we keep dividing. 12=2×6, so any multiple of 12 is a multiple of 2 and
therefore even. Since 2,375 is not even, 2,375 is not
23×3=69, so 23 can go into 83 at least 3
8 divisible by 12.
3 times. We subtract 69 from 83 and 5
23 198 2,375 ends in 75, so 2,375 is divisible by 25.
have 14 left over. Since 14 is less than
−115 50=2×25, so any multiple of 50 is a multiple of 2 and
23, we can’t subtract any more 23’s. 83
therefore even. Since 2,375 is not even, 2,375 is not
All together, we subtracted 3+5=8 −69
14 divisible by 50.
23’s.
2,375 does not end in 000, so 2,375 is not divisible by
So, the quotient of 198÷23 is 8 and the remainder is 14. 1,000.

12. 9,570 is somewhat close to 10,000, and 47 is close to 50. After circling the numbers that 2,375 is divisible by, we
So, we estimate that 9,570÷47 is about 10,000÷50=200. have

47×200=9,400. So, 47 can go into 200 2 4 5 12 25 50 50 1,000


9,570 at least 200 times. We subtract 47 9,570 17 16 20
−9,400 15. We know 4 is between 4 =4 and 4 =5.
9,400 from 9,570 and have 170 left 17 16
170 is one fourth more than 4 =4. So, written as a mixed
over. Since 170 is more than 47, we 4
17 1 1
keep dividing. number, 4 =4+ 4 =4 4 .
3 We count one fourth past 4 to reach 4 4 = 4 .
1 17
47×3=141, so 47 can go into 170 at 203
200
least 3 times. We subtract 141 from 47 9,570 4 5 6
170 and have 29 left over. Since 29 −9,400
is less than 47, we can’t subtract any 170
more 47’s. −141 17 1
29 4
=4 4
3 4
All together, we subtracted 16. Three sevenths is less than four sevenths. So, 7 < 7
.
200+3=203 forty-sevens.
17. Elevenths are greater than thirteenths. So, 3 elevenths are
So, the quotient of 9,570÷47 is 203 and the remainder is 3 3
greater than 3 thirteenths: 11 > 13.
29.
5 10 5
13. A number is divisible by 4 if and only if the number formed 18. We compare 17 to 37 by converting 17 to an equivalent
by its last two digits is a multiple of 4. The numbers whose fraction with numerator 10.
last two digits form a multiple of 4 are 300, 360, 600, 708, ×2
and 1,000. 5 10
17 = 34
A number is divisible by 25 if and only if its last two digits
10 10 5 10 ×2
are 00, 25, 50, or 75. The numbers whose last two digits > ,
34 37
so 17 > 37
.
are 00, 25, 50, or 75 are 275, 300, 600, 625, and 1,000.
A number is divisible by 100 if and only if its last two digits or
are 00. The numbers whose last two digits are 00 are 300, 5 10
You may have converted 17 and 37 to equivalent fractions
600, and 1,000. with the same denominator: 17×37=629.
After circling the numbers divisible by 4, boxing the
×37 ×17
numbers divisible by 25, and underlining the numbers 5 185 10 170
divisible by 100, we have 17 = 629
and
37 = 629
×37 ×17
187 275 300 360 535 600 625 708 1,000
185 170 5 10
>
629 629
, so 17 > 37
.

© 2014, AoPS Incorporated.


Are you ready for Beast Academy 4C?
19. First, we look for ways to make each product using two
different numbers from 1 to 8:
10=2×5 8=1×8 or 2×4 24=3×8 or 4×6
2=1×2 24=3×8 or 4×6 20=4×5
Since so many of the rows and columns have two possible
products, we begin this problem by trying to sort out which
products will be used.
We record the possible products on our puzzle as shown
below.
1×8 3×8
or or
2×5 2×4 4×6
10 8 24
1×2 2

3×8 or 4×6 24
4×5 20
Since the fourth row’s product can only be 20=4×5,
neither 4 nor 5 can be in any other row. So, in the third row,
we must use 24=3×8.
Since the first column’s product can only be 10=2×5,
neither the 2 nor 5 can be in any other column. So, in
the second column, we must use 8=1×8. Then, neither
the 1 nor 8 can be in any other column. So, we must use
24=4×6 in the fourth column.
1×8 3×8
or or
2×5 2×4 4×6
10 8 24
1×2 2

3×8 or 4×6 24
4×5 20

Each of the numbers from 1 to 8 must appear once in the


puzzle. From these products, we see that the product of the
unlabeled third column must be 3×7=21, and the product
of the unlabeled third row must be 6×7=42.
Now, each number is listed in its row and column, so we
complete the puzzle as shown.
1×8 3×8
or or
2×5 2×4 3×7 4×6
10 8 21 24
1×2 2 2 1
6×7 42 7 6
3×8 or 4×6 24 8 3
4×5 20 5 4

© 2014, AoPS Incorporated.


Are you ready for Beast Academy 4D?

Before beginning
Beast Academy 4D,
a student should be able to
add and subtract like fractions, A student
have a basic understanding of ready for Beast
negative integers, and know Academy 4D should
several basic counting be able to answer at
techniques. least 14 of the 19
problems below
correctly.

Step 1. The student should try to answer every question without a calculator and without help.
Step 2. Check the student’s answers using the solutions at the end of this document.
Step 3. The student should be given a second chance on problems that he or she answered incorrectly.

1. Fill in the blanks to create a four-digit 2. Fill in the blank to create a four-digit
number that is divisible by 2, 5, and 9. number that is divisible by 3 and 4.

8_3_ 247_

3. List all the factors of 196. 3. ______________________________

4. Write the prime factorization of 1,350. 4. ______________________________


Use exponents for repeated factors.

Compute each sum or difference below. Write your answer in simplest form, using
whole or mixed numbers when possible.

6 4
5. − = 6. 7 1 −1 3 =
7 7 4 4

2
7. 8 −3 12= 8. 5 14+3 6
=
13 13 17 17

9. How many centimeters are in the perimeter of an equilateral 9. ________


triangle whose sides are each 12 7 cm long?
10

© 2015, AoPS Incorporated.


Are you ready for Beast Academy 4D?

Fill in the missing numerators and denominators to make equivalent fractions.

1 25 14 700
10. = = 11. = =
4 1,000 20 10

Complete each skip-counting pattern.

12. 15, 11, 7, _______, _______, _______, _______, _______, _______

13. -20, -17, -14, _______, _______, _______, _______, _______, _______

Fill in the missing entries in the grids below to make all the equations true.

14. 9 + -2 =
15. 16 + =
− + − − − −
-3 − -5 = + 5 =
= = = = = =
+ = + -6 = -8

16. How many numbers are in the list below? 16. ________

115, 118, 121, ... , 247, 250, 253

17. How many different arrangements of the letters in the word FIONA 17. ________
are possible, including F-I-O-N-A?

18. How many three-digit numbers have an odd hundreds digit, an even 18. ________
tens digit, and an odd ones digit?

19. Six monsters compete in a handball tournament. Each monster


competes in exactly one game against each other monster. How 19. ________
many games are played?

© 2015, AoPS Incorporated.


Are you ready for Beast Academy 4D?
Solutions
1. Every multiple of 5 ends in 0 or 5. So, the ones digit of 6. 3
We count up. From 1 4 to 2 is 4 . Then, from 2 to 7 4 is
1 1
8_3_ is either 0 or 5. However, we also know that 8_3_ is 1
divisible by 2 (even), so 8_3_ cannot end in 5. 5 4 more.
The ones digit is 0. 3 1
14 2 3 4 5 6 7 74
Now, we have 8_30. For 8_30 to be divisible by 9, the
sum of its digits must be a multiple of 9. The sum of the
known digits is 8+3+0=11. The only multiple of 9 that we 1 1
54
can get by adding a digit (from 0 to 9) to 11 is 4
11+ 7 =18. Therefore, the hundreds digit is 7. 2
54
The four-digit number is 8730.
1 1 2 1
2. For 247_ to be divisible by 3, the sum of its digits must So, the difference equals 4 +5 4 =5 4 =5 2 .
be a multiple of 3. The sum of the known digits is or
2+4+7=13. The multiples of 3 that we can get by adding 3 1 1 1 3
a digit (from 0 to 9) to 13 are 13+ 2 =15, 13+ 5 =18, 1 4 is 4 less than 2. We add 4 to both 7 4 and 1 4 to make
and 13+ 8 =21. So, the ones digit is either 2, 5, or 8. the subtraction easier.
Every multiple of 4 ends in a two-digit multiple of four 1 3
7 4 −1 4
(00, 04, 08, 12, etc.) Since 247_ has a 7 in the tens place
and is divisible by 4, we know that it must end in either 72 2 3 4 5 6 7
or 76. So, the ones digit is either 2 or 6.
Only a ones digit of 2 meets both requirements, so the 2
7 4 −2
four-digit number is 2472.
1 3 2 2 1
3. We begin by looking at the product pairs with the smallest So, 7 4 −1 4 is equal to 7 4 −2=5 4 =5 2 .
factors and organize our work by writing the smaller
number in each pair !rst. or
3 1
196: To subtract 1 4, we can take away 2 and give back 4 .
1×196 1 3 1 1
2×98 7 4 −1 4 =7 4 −2+ 4
4×49 1 1
=5 4 + 4
7×28
14×14 2
=5 4
So, the factors of 196 are 1, 2, 4, 7, 14, 28, 49, 98, 1
=5 2 .
and 196.
4. We use a factor tree to organize our work. We circle the 7. We cannot subtract 12 thirteenths from 2 thirteenths, so
prime factors. 2 12 15 12 3
1,350 we regroup: 8 13−3 13=7 13−3 13=4 13.
We could have instead used one of the strategies
10 135 described in the previous problem to arrive at the same
difference.
2 5 5 27 14 6 20 3
8. 5 17+3 17=8 17=9 17.

3 9 9. The perimeter of an equilateral triangle with side length


7
12 10 cm is
7 7 7 21 1 1
3 3 12 10+12 10+12 10=36 +10=36 +2 10 =38 10 cm.

Writing these factors in order from least to greatest and 10. We can multiply the numerator and denominator of a
using exponents for repeated factors, we have fraction by the same number to make an equivalent
1,350=2×33×52. fraction. ×25 ×10
You may have used a different factor tree to arrive at the 1 25 250
same 3nal prime factorization. 4 = 100 = 1,000
×25 ×10
5. Subtracting 4 sevenths from 6 sevenths leaves us with
6 4 2
6−4=2 sevenths: 7 − 7 = 7 .

© 2015, AoPS Incorporated.


Are you ready for Beast Academy 4D?
11. We can multiply or divide the numerator and denominator Dividing each number in this new list by 3 gives us a list
of a fraction by the same number to make an equivalent from 1 to 47.
fraction. ÷2 ×100
3, 6, 9, …, 135, 138, 141
14 7 700 ÷3 ÷3 ÷3 … ÷3 ÷3 ÷3
20 = 10 = 1,000 1, 2, 3, …, 45, 46, 47
÷2 ×100
12. Each term in the pattern is 4 less than the one before it. This list is the same size as the original, and the numbers
−4 −4 are counted for us!
15, 11, 7, __, __, __, __, __, __ So, there are 47 numbers in the original list.
We continue the pattern by subtracting 4’s. 17. We begin by choosing one letter to be the #rst. There are
−4 −4 −4 −4 −4 −4 −4 −4
5 choices for the #rst letter (F, I, O, N, or A).
Then, no matter which letter we chose #rst, there are −
15, 11, 7, 3, -1, -5, -9, -13, -17
always 4 choices for the second letter.
13. Each term in the pattern is 3 more than the one before it.
Similarly, no matter which two letters we chose #rst and
+3 +3
second, there are always 3 choices remaining for the third
-20, -17, -14, __, __, __, __, __, __ letter.
We continue the pattern by adding 3’s. Next, no matter which three letters we chose #rst, second,
+3 +3 +3 +3 +3 +3 +3 +3 and third, there are always 2 choices remaining for the
fourth letter.
-20, -17, -14, -11, -8, -5, -2, 1, 4
Finally, once we have chosen the #rst four letters, there is
only 1 remaining letter to be #fth.
14. Step 1: Final:
All together, there are 5×4×3×2×1=120 ways to arrange
9 + -2 = 7 9 + -2 = 7 the #ve letters in some order. You may have also written
− + − − + − this as 5! or “5 factorial.”
-3 − -5 = 2 -3 − -5 = 2 or
= = = = = =
We consider placing the letters in these blanks: _ _ _ _ _ .
12 + -7 = 12 + -7 = 5
There are 5 possible spots we could choose for the F. No
matter where F is placed, there are 4 remaining spots we
15. Step 1: Step 2:
could choose for I. No matter where F and I are placed,
16 + -1 = 16 + -1 = 15 there are 3 remaining spots we could choose for O. No
− − − − − − matter where F, I, and O are placed, there are 2 remaining
+ 5 = 18 + 5 = spots we could choose for N. Finally, after placing the
= = = = = = other four letters, there is just 1 spot remaining for the A.
-2 + -6 = -8 -2 + -6 = -8 All together, there are 5×4×3×2×1=120 ways to arrange
the four letters in some order. You may have also written
Final:
this as 5! or “5 factorial.”
16 + -1 = 15
18. There are #ve odd digits: 1, 3, 5, 7, and 9.
− − − There are #ve even digits: 0, 2, 4, 6, and 8.
18 + 5 = 23
So, we have 5 choices for the odd hundreds digit, 5
= = = choices for the even tens digit, and 5 choices for the odd
-2 + -6 = -8 ones digit.
All together, we can create 5×5×5=125 such numbers.
16. This list counts by 3’s, but the numbers are not all 19. Each game is played by one pair of monsters. When
multiples of 3. To make the list easier to count, we choosing a pair of monsters, we have 6 choices for the
subtract 112 from each number. This gives us a list of all “#rst” monster and 5 choices for the “second” monster.
multiples of 3 from 3 to 141:
However, since “Grogg vs. Lizzie” is the same game
115, 118, 121, …, 247, 250, 253 as “Lizzie vs. Grogg,” the order in which we pick the
−112 −112 −112 … −112 −112 −112 monsters does not matter. So, when we multiply 6×5 to
3, 6, 9, …, 135, 138, 141 get 30, we have counted each game twice.
Therefore, there are 30÷2=15 games in the tournament.

© 2015, AoPS Incorporated.


Are you ready for Beast Academy 5A?

Before beginning
Beast Academy 5A,
a student should be familiar
with variables, adding and A student
subtracting integers and ready for Beast
decimals, and multiplying Academy 5A should
whole numbers with be able to answer at
fractions. least 14 of the 18
problems below
correctly.

Step 1. The student should try to answer every question without a calculator and without help.
Step 2. Check the student’s answers using the solutions at the end of this document.
Step 3. The student should be given a second chance on problems that he or she answered incorrectly.

Evaluate each sum, difference, or product below.

1. 1.234+9.87 = _______ 2. 5.15−1.515 = _______

5 3
3. 12
×9= _______ 4. 7×11 8 =_______

5. 7+8−9−10= _______ 6. -4−(-3)+(-2)−(-1) = _______

3
7. What is the perimeter of a regular hexagon with side length 4 4 ft? 7. _________

8. What is the probability that a randomly selected factor of 36 is odd? 8. _________

Fill the missing cells of each grid below so that each row,
column, and diagonal has the sum given.

9. Sum: 5.4 10. Sum: -6

0.57 5 -4

1.8

0.77 3

© 2015, AoPS Incorporated.


Are you ready for Beast Academy 5A?

Find the value of the variable in each equation below.

11. 5×w=37 12. s+33=16 11. w=_____

12. s=_____

Evaluate each expression below for a=6 and b=3.

13. 7×a+18÷(a+b) 14. 12−b×7×(10−a) 13. ________

14. ________

15. Use the given digits once each 16. Trace a path through the grid that
to fill the blanks in the statement crosses every number in order from
below so that the statement is true. least to greatest.

1, 3, 4, 5 0.05 0.50 5.05

5 4 < 3 < 5 50.55 50.50 0.55 5.50

55.05 50.05 5.55

17. Five congruent rectangles are attached as shown. What is 17. Perimeter: ________
the perimeter and area of the larger rectangle they create?
Area: ________

22 cm

18. Rita picks a number from the list below at random. 18. ________

10, 11, 12, ... , 77, 78, 79

What is the probability that her number is not divisible by 5?

© 2015, AoPS Incorporated.


Are you ready for Beast Academy 5A?
Solutions
1. We write 9.87 as 9.870 so that both both numbers have 8. The factors of 36 are 1, 2, 3, 4, 6, 9, 12, 18, 36.
digits with the same place values. Then, we stack the Among the nine factors of 36, three are odd. So, the
numbers vertically and add the thousandths, hundredths,
probability that a randomly selected factor of 36 is odd is
tenths, and ones.
# of odd factors of 36 3 1
1 1 1 1 1
# of factors of 36
=9=3.
1.234 1.234 1.234 1.234
+ 9.870 + 9.870 + 9.870 + 9.870 9. In the middle column, we have 0.57+1.8=2.37. So, the
4 04 104 11.104 bottom cell in the middle column is 5.4−2.37=3.03.
Similarly, in the diagonal from top-left to
2. We write 5.15 as 5.150 so that both numbers have
bottom-right, we have 1.8+0.77=2.57. So, the
digits in the same place values, then stack the numbers
top-left cell is 5.4−2.57=2.83.
vertically and subtract the thousandths, hundredths,
tenths, and ones as shown below.
2.83 0.57
4 10 4 10 4 11 4 10 4 11 4 10
5.150 5.150 5.150 5.150
− 1.515 − 1.515 − 1.515 − 1.515 1.8
5 35 635 3.635
3.03 0.77
5 5×9 45 15 3
3. 12
×9= 12 =12= 4 =3 4 .
We use the same method to fill the remaining cells as
or
shown in the steps below:
We notice both 9 and 12 are multiples of 3:
2.83 0.57 2 2.83 0.57 2
5 5×9 9 3 15 3
12
×9= 12 =5×12=5× 4 = 4 =3 4 .
1.8 0.97 1.8 2.63
3 3 91
4. We convert 11 8
to a fraction: 11 8 = 8 .
1.6 3.03 0.77 1.6 3.03 0.77
91 91×7 637 5
Then, we multiply: 8 ×7= 8 = 8 =79 8 .
or
10. In the top row, we have 5+(-4)=1. So, the top cell in the
We use the distributive property: middle column is -6−1=-7.
3
7×11 8 =7× 11+ 8 )( 3
5 -7 -4
=(7×11)+(7× 8 )
3

21
=77+ 8
5 3
=77+2 8
5
=79 8 . We use the same method to fill the remaining cells as
shown in the steps below:
5. We add and subtract, from left to right:
7+8=15, then 15−9=6, and 6−10=-4. 5 -7 -4 5 -7 -4 5 -7 -4

6. -4−(-3)+(-2)−(-1)=-4+3+(-2)+1=-2. -2 -2 -11 -2 7
7. The perimeter of a regular hexagon with side length 3 0 3 -9 0 3 -9
3
4 4 feet is

( ) ( ) If 5×w=37, then w=37÷5= 5 =7 5 .


1 37 2
3 3 18
6×4 4 = 6×4 + 6× 4 =24+ 4 =24+4 4 =28 4 =28 2 ft.
2 2 11.
or 37 5×37 5
We check that 5× 5 = 5 = 5 ×37=1×37=37. ü
3
6×4 4 = (6×4)+( 3
6× 4 ) 18 9 1 1
=24+ 4 =24+ 2 =24+4 2 =28 2 ft. 12. If s+33=16, then s=16−33=-17.
We check that -17+33=16. ü

© 2015, AoPS Incorporated.


Are you ready for Beast Academy 5A?
13. Replacing a with 6 and b with 3, we get 18. We first count the numbers in Rita’s list. To make the list
easier to count, we subtract 9 from each number. This
7×6+18÷(6+3).
gives us a list from 1 to 70.
Then, using the order or operations to evaluate this
expression gives us 10, 11, 12, …, 77, 78, 79
−9 −9 −9 … −9 −9 −9
7×6+18÷(6+3)=7×6+18÷9=42+2=44.
1, 2, 3, …, 68, 69, 70
14. Replacing a with 6 and b with 3, we get
So, there are 70 numbers in the original list.
12−3×7×(10−6).
Next, we count the numbers in the original list which are
Then, using the order or operations to evaluate this divisible by 5:
expression gives us 10, 15, 20, ... , 65, 70, 75.
12−3×7×(10−6)=12−3×7×4=12−84=-72. To make this list easier to count, we divide each number
15. Since the ones digits of the first and last numbers are both by 5 to get
5, the ones digit of the middle number is also 5. 2, 3, 4, ..., 13, 14, 15.
5 4< 5 3 < 5 Subtracting 1 from each number in this list gives us a list
of whole numbers from 1 to 14.
Then, the number on the far right must have tenths digit
larger than 3. The only remaining digit larger than 3 is 4. So, there are 14 multiples of 5 in this list, leaving
70−14=56 numbers that are not divisible by 5.
5 4< 5 3 < 5 4
Therefore, the probability that Rita’s number is not
Finally, only placing the 1 and 3 into the remaining empty divisible by 5 is
boxes as shown creates a true statement. # of #s not divisible by 5 in list 56 4
# of #s in list
= 70 = 5 .
5 1 4< 5 3 3 < 5 4
or
16. We begin by circling the smallest number on the grid,
0.05. Then, we move from hexagon to hexagon, always There are 70 numbers in this list, and 14 are divisible by
connecting to the next-smallest number. We finish at the 5. So, the probability that Rita chooses a number that is
largest number, 55.05. divisible by five is
# of #s divisible by 5 in list 14 1
0.05 0.50 5.05 # of #s in list
= 70 = 5 .

50.55 50.50 0.55 5.50 Therefore, the probability that Rita’s choice is not divisible
1 4
by five is 1− 5 = 5 .
55.05 50.05 5.55

17. The length of the short side of each congruent rectangle is


22 11 1
22÷4= 4 = 2 =5 2 cm.
1
5 2 cm
1
5 2 cm
1
22 cm
5 2 cm
1
52 cm

So, the height of the large rectangle is 22 cm and its width


1 1
is 22+5 2 =27 2 cm. 1
22 52

22 cm

1
27 2 cm
Therefore, the perimeter of the large rectangle is
( 1
(22+22)+ 27 2 +27 2 =44+55=99 cm.
1
)
The area of the large rectangle is
1 55 22
22×27 2 =22× 2 =55× 2 =55×11=605 sq cm.
© 2015, AoPS Incorporated.
Are you ready for Beast Academy 5B?

Before beginning
Beast Academy 5B, a
student should be able to
order fractions, compute with A student
negative integers, find prime ready for Beast
factorizations, and write and Academy 5B should
solve basic equations be able to answer at
using variables. least 12 of the 16
problems below
correctly.

Step 1. The student should try to answer every question without a calculator and without help.
Step 2. Check the student’s answers using the solutions at the end of this document.
Step 3. The student should be given a second chance on problems that he or she answered incorrectly.

1. Order the fractions below from least to greatest. 1. ___________________________


1 2 4 1 4 7
5 11 3 4 7 6
2. Compute the perimeter in meters of the rectangle below. 2. _________
7
28 m

3
18 m

3. List all of the factors of 42 that are also multiples of 3. 3. _____________________

4. What is the largest two-digit integer whose prime factorization 4. _________


includes exactly three 3’s?

Evaluate each expression below.


8−2 (-1)15 • 3+15
5. 7+3(-6)=____ 6. =____ 7. -5+ =____
-4+1 -2(-3)

8. Circle the expressions below that are equal to -25.

-52 (-5)2 -(52) -(-5)2 (-52) -(-52)

3ab−4
9. Evaluate when a=-4, b=5, and c=-16. 9. _________
-c

© 2016, AoPS Incorporated.


Are you ready for Beast Academy 5B?

Find the value of the variable in each equation below.

3+n
10. =9 11. 47+36k=32k+95 10. n=_____
4
11. k=_____

12. In the Circle Sum puzzle below, the number in each circle is the sum 12. t=_____
of the numbers in the connected circles below it. What is t?
40

4t t -2

13. The combined cost of a mop and a jumpsuit is 39 dollars. If the 13. _________
jumpsuit costs 3 dollars more than 5 times the cost of the mop, then
how much does the jumpsuit cost?

14. A pyramid has exactly 12 edges. What is the shape of its base? 14. _________

15. The solid below was created by attaching 6 unit cubes. What is the 15. _________
surface area of the solid?

16. A triangular prism is attached to the top face of a cube as shown. 16. _________
What is the volume in cubic inches of the new solid?
4
in
in
3

5 in

© 2016, AoPS Incorporated.


Are you ready for Beast Academy 5B?
Solutions
1
1. There are three fractions in the list that are less than 2
: -5+
(-1)15 • 3+15 -1 • 3+15
=-5+ -2(-3)
1 2 1 1 -2(-3)
, , and 4 . Since fifths are smaller than fourths, 5 is less
5 11 -3+15
=-5+ 6
1 2 2 1 2
than 4 . Then, 11 is less than 10= 5 . So, 11 is the smallest 12
1 1 =-5+ 6
fraction in the list, followed by 5 , then by 4 .
4 1 =-5+2
Of the remaining fractions, 7 is greater than 2 but less
4 7 4 =-3.
than 1, while 3 and 6 are greater than 1. So, 7 is the next
fraction in the list. 8. We evaluate each expression:
-52=-(5 • 5)=-25.
We write the remaining two fractions as mixed numbers:
(-5)2=(-5) • (-5)=25.
4 1 7 1 1 2 1
3
=1 3 and 6 =1 6 . Since 3 = 6 is greater than 6 , we see -(52)=-(5 • 5)=-25.
4 7
that 3 is greater than 6 . -(-5)2=-((-5) • (-5))=-(25)=-25.
(-52)=(-(5 • 5))=-25.
So, the list of fractions in order from least to greatest is
2 1 1 4 7 4 -(-52)=-(-(5 • 5))=-(-25)=25.
, , , , , .
11 5 4 7 6 3
So, the four circled expressions are equal to -25.
2. The perimter of the rectangle is
-52 (-5)2 -(52) -(-5)2 (-52) -(-52)
2 8 +1 8 +2 8 +1 8 =(2+1+2+1)+( 8 + 8 + 8 + 8 )
7 3 7 3 7 3 7 3

20
=6+ 8 9. When a=-4, b=5, and c=-16, we have
=6+2 8
4 3ab−4 3(-4)(5)−4
-c
= -(-16)
4 (-12)(5)−4
=8 8 = 16
1 -60−4
=8 2 meters. = 16
3. The factors of 42 are 1, 2, 3, 6, 7, 14, 21, and 42. Of these -64
= 16
factors, only 3, 6, 21, and 42 are also multiples of 3.
4. A number that has exactly three 3’s in its prime =-4.
factorization must be a multiple of 33=27. The two-digit 10. In this equation, we add 3 to n, then divide the result by
multiples of 27 are 27, 54, and 81 (the next multiple is 4 to get 9. To isolate n, we undo these steps in reverse
108, which has three digits). order.
Of these numbers, 81 is the largest. However, the prime To undo dividing by 4, we multiply both sides of the
factorization of 81 is 34, and we are told that the prime 3+n
equation by 4. Since 4 • 4 simplifies to 3+n, we have
factorization of the number has exactly three 3’s.
3+n
The next-largest number is 54, which has prime • 4=9 • 4
4
factorization 2 • 33. Therefore, the largest two-digit number
with exactly three 3’s in its prime factorization is 54. 3+n=36
5. Following the order of operations, we compute the product Then, to undo adding 3, we subtract 3 from both sides of
3(-6), then add 7 to the result: 3+n=36, giving n=33.
7+3(-6)=7+(-18) 11. We first get all terms with k on one side of the equation.
=-11. We can eliminate 32k from the right side of the equation
by subtracting 32k from both sides.
6. Following the order of operations, we evaluate the
numerator and denominator first, then divide: 47+36k = 32k+95
−32k −32k
47 + 4k = 95
8−2 6
= =-2.
-4+1 -3

7. We first simplify the numerator and denominator of the Then, subtracting 47 from both sides of 47+4k=95 gives
fraction, then divide, then add. 4k=48. Dividing both sides by 4, we have k=12.

In the numerator, (-1)15 is the product of an odd number of


-1’s, so its result is negative: (-1)15=-1. So, we have

© 2016, AoPS Incorporated.


Are you ready for Beast Academy 5B?
12. Each circle is labeled with the sum of the numbers below 16. The volume of the new solid is the sum of the volumes of
it. So, we label the left blank circle 4t+t=5t and the right the cube and the triangular prism.
blank circle t+(-2)=t−2.
A cube with side length 5 in has volume 5 • 5 • 5=125
40 cubic inches.
To compute the volume of the triangular t
5t t−2 igh
prism, we multiply the area of its base by He
3•4 4
its height. The area of the base is 2 =6 in

in
4t t -2 square inches. The height of the prism is Base

3
n
5i
5 inches. So, the volume of the triangular
Then, we use the top three circles to write an equation: prism is 6 • 5=30 cubic inches.
5t+(t−2)=40. Therefore, the total volume of the solid is 125+30=155
Combining like terms, the equation simplifies to cubic inches.
6t−2=40.
Adding 2 to both sides gives 6t=42. Dividing both sides
by 6, we have t=7.
13. We let m represent the cost of a mop in dollars. Since a
jumpsuit costs 3 dollars more than 5 times the cost of a
mop, the jumpsuit costs 5m+3 dollars.
The combined cost of a mop and a jumpsuit is 39 dollars.
So, we can write an equation:
m+(5m+3)=39.
Combining like terms, the equation simplifies to
6m+3=39.
Subtracting 3 from both sides gives 6m=36. Dividing both
sides by 6, we have m=6.
Since m represents the cost of a mop, the mop costs 6
dollars. Since 39 dollars is the cost of the mop and the
jumpsuit, the jumpsuit costs 39−6=33 dollars.
14. If there are n edges on the base of the pyramid, then there
are n edges that connect the base to the apex, which give
us a total of n+n=2 n edges.
The number of edges is two times the number
of sides on its base, so a pyramid with exactly
12 edges has a base with 12÷2=6 sides. A
6-sided polygon is a hexagon.
15. We can count the number of visible faces on the outside
of the solid from each of the six views.
Front: 5 faces Top: 5 faces Right: 3 faces

Back: 5 faces Bottom: 5 faces Left: 3 faces

Each face on each small cube is a unit square with


area 1. So, the total surface area of the solid is
5+5+3+5+5+3=26 square units.

© 2016, AoPS Incorporated.


Are you ready for Beast Academy 5C?
Before
beginning Beast A student
Academy 5C, a ready for Beast
The Academy 5C should
student should be able student should
to compute fluently with be able to answer at
also have a basic least 12 of the 16
fractions and integers understanding of
and be able to add problems below
statistics, greatest correctly.
and subtract common factor, and
decimals. least common
multiple.

Step 1. The student should try to answer every question without a calculator and without help.
Step 2. Check the student’s answers using the solutions at the end of this document.
Step 3. The student should be given a second chance on problems that he or she answered incorrectly.

Evaluate each expression below.

1. 4.372+11.91=_______ 2. 8.36−1.058=_______

3. 3 56 + 2 10
3
=____ 4. 4 12 − 35 =____

5 12
5. 18 35 =____
• 6. 3 47 ÷ 58 =____

7. Order the numbers below from least to greatest. 7. ______, ______, ______, _______
3 437
4 10 4.037 100 4.307

8. a. What is the greatest common factor (GCF) of 48 and 90? a. GCF: _______

b. What is the least common multiple (LCM) of 48 and 90? b. LCM: _______

9. a. What is the GCF of 36, 54, and 60? a. GCF: _______

b. What is the LCM of 36, 54, and 60? b. LCM: _______


© 2016, AoPS Incorporated.
Are you ready for Beast Academy 5C?

For problems 10-12, use the given numbers to fill in the blanks so that
• each statement is true, and
• each fraction is in simplest form.

10. Numbers: 3, 5, 9, 20 11. Numbers: 3, 5, 6, 14 12. Numbers: 5, 12, 20, 30

4 3 17
• = • = + =
6 8 35

Use the prime factorization below to help you answer problems 13 and 14.
159,600=24 • 3 • 52 • 7 • 19

13. What is the smallest positive integer that we can multiply 13. ________
159,600 by to get a product that is a perfect square?

14. What is the smallest positive integer that is not a factor of 159,600? 14. ________

15. A wheelbarrow contains five 6-pound pumpkins and some 19-pound 15. ________
pumpkins. If the average weight of a pumpkin in the wheelbarrow is
14 pounds, how many 19-pound pumpkins are in the wheelbarrow?

16. Fill each empty white square below with a positive digit so that the clues given in the
surrounding shaded squares give the correct average, mode, median, and range for the
row or column they label.
average
7 5
2 6
range

mode

4
8
7
median

© 2016, AoPS Incorporated.


Are you ready for Beast Academy 5C?
Solutions
1. 4.372+11.91=16.282. b. To compute the LCM, we take the largest power of
2. 8.36−1.058=7.302. each prime in the numbers’ prime factorizations.
5 3 25 9 34 4 2 92 36=22 • 32 54= 2 • 33 60= 22 • 3 • 5
3. 3 6 + 2 10=3 30 + 2 30=5 30=6 30=6 15=15.
So, the LCM of 36, 54, and 60 is 22 • 33 • 5=540.
1 3 5 6 15 6 9 39
4. 4 −
2 5
=4 10 − 10=3 10 − 10=3 10=10.
10. All fractions must be in simplest form. Among 5
• =8
5. We first divide 5 and 35 by their 1
5 12
our choices, only 5 can be the numerator of 6

greatest common factor, 5. 18 35 the fraction with denominator 6.
7
We then divide 12 and 18 by their 1 2 The remaining numbers are 3, 9, and 20.
5 12

greatest common factor, 6. 18 35 Since the denominator of the product 3 or 9
3 7
is not a multiple of 3, the factor of 3 in 5
Now that we have canceled all 1 2
2 5 • =8
5

12 1 • 2
= = the denominator of 6 must cancel. So, 6
common factors, we compute the 18 35 3 • 7 21
3 7 the numerator of the middle fraction is a
product.
5 multiple of 3.
4 25 4 5 25 5 25 8 25 8 40 5
6. 3 7 = 7 . So, 3 7 ÷ 8 = 7 ÷ 8 = 7 • 5 = 7 • 5 = 7 =5 7 . Among our choices, only 3 and 9 are
1
multiples of 3.
7. We write each fraction as a decimal.
Since all fractions are in simplest form, 20 3 or 9
3 437
4 10=4.3 4.037 100
=4.37 4.307 cannot be the numerator of 8 . Therefore, 20 5
• =
Then, we compare the decimals: is the denominator of the middle fraction. 6 20 8

4.037 < 4.3 < 4.307 < 4.37. Finally, we place 3 and 9 in the empty 5 3 9
• =
6 20 8
Writing the original numbers in order from least to numerators in the only way that makes a
greatest, we have true statement.
3 437 1 3
4.037, 4 10 , 4.307, 100 . 5 9 1•3 3
Check: 6 • 20=2 • 4= 8 . ü
or 2 4

We write each number as a mixed number with a 11. The denominator of the product is 35=5 • 7.
denominator of 1,000. So, the denominators of the two fractions
3 300 37 that we multiply must include at least one
4 10=4 1,000 4.037=4 1,000
multiple of 5 and one multiple of 7.
437 37 370 307 Among our choices, only 5 is a multiple of
=4 100 =4 1,000 4.307=4 1,000
100 5, and only 14 is a multiple of 7. These two
Then, we compare the mixed numbers: numbers are therefore the denominators of
37 300 307 370 the fractions that we multiply.
4 1,000 < 4 1,000 < 4 1,000 < 4 1,000 .
Since all fractions are in simplest form, 14 4
• =
Writing the original numbers in order from least to 4 14 5 35
cannot be the denominator of . We place
greatest, we have
3 437 5 and 14 in the empty denominators as
4.037, 4 10 , 4.307, 100 . shown.
8. a. We use the prime factorizations of 48 and 90. The remaining numbers are 3 and 6. 3 4
• =
6
14 5 35
48=2 4 •
3 and 90= 2 3 • 2 •
5 All fractions are in simplest form, so we
The GCF of 48 and 90 is the product of all the prime place the 3 and 6 in the empty numerators
factors they share. So, the GCF is 2 • 3=6. as shown.
2
b. To compute the LCM, we take the largest power of 3 4
Check: 14 • 5 =7 • 5=35. ü
3•2 6
each prime in either number’s prime factorization. 7

48=24 • 3 and 90= 2 • 32 • 5 12. Both the numerator and denominator of the 5 and 12
So, the LCM of 48 and 90 is 24 • 32 • 5=720. middle fraction are empty.
3 17
All fractions are in simplest form. Among + =
9. a. We use the prime factorizations of 36, 54, and 60.
our four number choices, 5 and 12 are the
36=22 • 32 54= 2 • 33 60= 22 • 3 • 5 only pair with a GCF of 1. So, only 5 and
The GCF of 36, 54, and 60 is the product of all the 12 can be the numerator and denominator
prime factors they share. So, the GCF is 2 • 3=6. of the middle fraction.

© 2016, AoPS Incorporated.


Are you ready for Beast Academy 5C?
The remaining numbers are 20 and 30. 5 and 12 However, 9=32 is not a factor 24 • 3 • 52 • 7 • 19. So, 9 is the
Since each fraction is in simplest form, smallest positive integer that is not a factor of 159,600.
3 17
we can only place 20 and 30 in the empty 20
+ = 30 15. Each 6-pound pumpkin is 8 pounds below the average,
denominators as shown. and each 19-pound pumpkin is 5 pounds above the
17 3 5 17 average. The five 6-pound pumpkins are a total of 5 •
Then, since the sum 30 is less than 1 and + =
20 12 30 8=40 pounds below the average. To balance this, we
we are adding only positive numbers, we need enough 19-pound pumpkins to equal 40 pounds
know the middle fraction is also less than above the average.
1. So, we place the 5 and 12 as shown.
−40 +40
3 5 9 25 34 17 −8 −8 −8 −8 −8 +5 ... +5
Check: 20 + 12=60 + 60=60=30. ü
6 6 6 6 6 19 ... 19
13. If a number is a perfect square, then it can be written as 40
So, there are 5 =8 nineteen-pound pumpkins in the
the product of two identical groups of prime factors. So, in
wheelbarrow.
the prime factorization of a perfect square, every prime has
an even exponent. or
In the prime factorization of 159,600=24 • 3 • 52 • 7 • 19, the We write and solve an equation. Let n be the number of
primes 3, 7, and 19 each have odd exponents (31, 71 and 19-pound pumpkins in the wheelbarrow.
191). So, multiplying 159,600 by 3 • 7 • 19 gives a perfect • The five 6-pound pumpkins weigh a total of 5 • 6=30
square: pounds, and the n 19-pound pumpkins weigh a total of
159,600 • 3 • 7 • 19=(24 • 3 • 52 • 7 • 19) • 3 • 7 • 19 19n pounds. So, the total weight of all the pumpkins is
= 24 • 32 • 52 • 72 • 192 30+19n.
=(22 • 3 • 5 • 7 • 19) • (22 • 3 • 5 • 7 • 19) • There are 5+n pumpkins in the wheelbarrow with an
=7,980 • 7,980 average weight of 14 pounds, so their total weight is
=7,9802. 14(5+n)=70+14n.
Multiplying 159,600 by any integer less than 3 • 7 • 19 would We have two expressions for the total weight of the
result in a product that is not a perfect square. So, pumpkins, so we write an equation:
3 • 7 • 19=399 is the smallest positive integer we can
30+19n=70+14n.
multiply 159,600 by to make a perfect square.
Solving for n, we get n=8. Therefore, there are 8
14. If an integer x is not a factor of 159,600, then either
19-pound pumpkins in the wheelbarrow.
• x has at least one prime factor that is not a factor of
159,600, or 16. Adding all of the numbers in a list average
• the power of a prime factor x is larger than its power in and then dividing by the number of 7 5
159,600. numbers gives us their average. 2 6
range

mode
The smallest prime factor that is not in 24 • 3 • 52 • 7 • 19 is 11. The average of the numbers in 4
The smallest power of 2 larger than 24 is 25=32. the left column is 7, so the sum of 8
The smallest power of 3 larger than 3 is 32=9. these three numbers is 3 • 7=21.
7
The smallest power of 5 larger than 52 is 53=125. There is already a 4 in this median
The smallest power of 7 larger than 7 is 72=49. column, so the sum of the two
The smallest power of 19 larger than 19 is 192=361. remaining numbers is 21−4=17. (8 and 9)
Among the possibilities above, the smallest is 32=9. The only way to make a sum of 17
So, 9 is the smallest positive integer that is not a factor of from two positive digits is 8+9.
159,600.
or The range is the difference average
between the largest and smallest 7 5
Each integer from 1 to 8 is a factor of 24 • 3 • 52 • 7 • 19:
numbers in a data set. 2 8 6
1: 24 • 3 • 52 • 7 • 19=1 • (24 • 3 • 52 • 7 • 19)
range

mode

In the top row, the range is 2. 4


2: 24 • 3 • 52 • 7 • 19=2 • (23 • 3 • 52 • 7 • 19) The top row already contains a 6,
3: 24 • 3 • 52 • 7 • 19=3 • (24 • 52 • 7 • 19) 9 8
so it cannot contain any digit larger
4: 24 • 3 • 52 • 7 • 19=4 • (22 • 3 • 52 • 7 • 19) than 6+2=8.
7
5: 24 • 3 • 52 • 7 • 19=5 • (24 • 3 • 5 • 7 • 19) median
6: 24 • 3 • 52 • 7 • 19=6 • (23 • 52 • 7 • 19) So, we place the 8 and 9 in the left
column as shown. (8 and 9)
7: 24 • 3 • 52 • 7 • 19=7 • (24 • 3 • 52 • 19)
8: 24 • 3 • 52 • 7 • 19=8 • (2 • 3 • 52 • 7 • 19)

© 2016, AoPS Incorporated.


Are you ready for Beast Academy 5C?
The mode is the number that average
appears most in a list. 7 5
The mode of the three integers 2 8 6

range

mode
in the bottom row is 8, so this 4
number must appear at least 9 8 8 8
twice in the row. Since we already
7
have a 9 in the bottom row, the
median
two other squares in this row both
contain 8.

The median is the number in the average


middle when we arrange a list in 7 5
order from least to greatest. 2 8 6
range

mode
The median of the middle column 4 7
is 7. Since this column already 9 8 8 8
contains a 6 and 8, the remaining
7
number in this column is 7.
median
The average of the numbers in the average
right column is 5, so the sum of 7 5
these three numbers is 3 • 5=15. 2 8 6
range

mode

There is already an 8 in this 4 7


column, so the sum of the two 9 8 8 8
remaining numbers is 15−8=7.
7
We can write 7 as the sum of two
median
digits in 3 ways: 1+6, 2+5, and (1 and 6),
3+4. (2 and 5),
or (3 and 4)

The range in the top row is 2, and average


the two digits already placed have 7 5
a range of 2. Therefore, the third 2 8 6 6
number in this top row can only be
range

mode

4 7 1
6, 7, or 8.
9 8 8 8
Only one of our three pairs in the
7
right column contains a 6, 7, or 8:
median
1+6. (1 and 6),
So, we place the 1 and 6 in the (2 and 7),
or (3 and 4)
right column as shown to complete
the puzzle.

© 2016, AoPS Incorporated.


Are you ready for Beast Academy 5D?

The student should


Before beginning also have a good
Beast Academy 5D, a understanding of prime A student
student should be able factorization. ready for Beast
to compute fluently with Academy 5D should
fractions, decimals, and be able to answer at
negative numbers. least 13 of the 18
problems below
correctly.

Step 1. The student should try to answer every question without a calculator and without help.
Step 2. Check the student’s answers using the solutions at the end of this document.
Step 3. The student should be given a second chance on problems that he or she answered incorrectly.

Convert each fraction below into a decimal.

3 5 4
1. 10 =______ 2. 8 =______ 3. 15 =______

Write each product below as a mixed number in simplest form.

7 70
4. 0.85 • 10=______ 5. 12 90=______
• 6. 3 0.1=______

Solve for each variable below. Write fractional answers in simplest form.

7.
3 15
=a a=________ 8. c =3 c=________
5 10 4

9. 2 x+6=x x=________ 10. z+1 +6=3 z=________


10

11. Three more than five times n is 7. What is n? 11. n=________

© 2017, AoPS Incorporated.


Are you ready for Beast Academy 5D?

Fill the blanks to complete each arithmetic sequence below. Write any terms that
are not whole numbers as mixed numbers.

12. 39, _____, _____, 53, _____, _____, 67, ...

13. 50, _____, 12, _____, _____, _____, -64, ...

Answer each of the questions below.

14. Fill in the blank below to make a true equation.


2
12 • 14 • 21 • 2=

15. What is the smallest positive integer that can be multiplied by 0.4 to 15. ________
get an integer result?

16. If we multiply all the terms in sequence below, will the result be 16. _____________
positive or negative?

1, -2, 3, -4, ..., 29, -30, 31.

17. What is the power of 5 in the prime factorization of the number 17. ________
represented by the product below?

5 • 10 • 15 • ... • 90 • 95 • 100

18. The ratio of Edgar’s age to Frank’s age is 3:6. The sum of their ages 18. ________
is 36. How many years old is Edgar?

© 2017, AoPS Incorporated.


Are you ready for Beast Academy 5D?
Solutions
3 17
1. 10
is 3 tenths. Written as a decimal, 3 tenths is 0.3. So, 0.85 • 10=20 • 10, and we have
You may have written .3. 17 17 1
• 10= =8 .
5 20 2 2
2. We wish to convert to an equivalent fraction with a
8
2

denominator that is a power of 10. Neither 10 nor 100 7 630 105 1


5. 12
• 90=
12
= 2 =52 2 .
is a multiple of 8. However, 8×125=1,000. So, we can
convert as shown below. or
15
×125 7 7 105 1
• 90= • 90= =52 2 .
12 12 2
5 625 2
=
8 1,000 1 70 70 1
×125 6. 0.1=10. So, 3 • 0.1= 3 • 10, and we have
5 625
So, 8 = 1,000 , which is 625 thousandths: 0.625. 70 1 70 1 7
7
1

3 10
= 3 • 10= 3 =2 3 .
You may have written .625.
3
or 7. We multiply the numerator and denominator of 5 by 5 to
5 get an equivalent fraction with numerator 15.
Since 8 means 5÷8, we use long division to write this
×5
number as a decimal.
0.005 3 15
=
0.02 0.02 0.625 5 25
0.6 0.6 0.6 ×5
8 5.0 8 5.0 8 5.0 3 15 3 15
−4.8 −4.8 −4.8 Since 5
=a and = ,
5 25
we have a=25.
0.2 0.20 0.20 or
−0.16 −0.16 We multiply both sides by 5a and then solve for a as
0.04 0.040
shown below.
−0.040 3 15
=a
0 5
3
5
So, 8 =0.625. You may have written .625. 5
•5 a=15
a • 5a
3. 4
Since 15 means 4÷15, we use long division to write this 3 • a=15 • 5
number as a decimal. 3a=75
0.006
0.06 0.06 a=75
3
0.2 0.2 0.2 a=25.
15 4.0 15 4.0 15 4.0
−3.0 −3.0 −3.0 8. We multiply both sides by 10 and then solve for c as
1.0 1.00 1.00 shown below.
−0.90 −0.90 c 3
=
10 4
0.10 0.100 c • 3 5
−0.090 10
10= 4 • 10
0.010 2

We notice a pattern! Each time we divide after the first


c=15
2
step, we get a remainder that is a decimal ending in “10”. c=7 12 .
When we divide these remainders by 15, the quotient is
always a decimal ending in 6. You may have instead written 7.5.
We usually write repeating decimals by drawing a bar
4
9. Subtracting x from both sides of the equation gives
over the digits that repeat. So, 15=0.2666...=0.26.
x+6=0
You may have written .26 , or you may have written
Then, subtracting 6 from both sides gives
0.2666... with three dots at the end indicating that the 6’s
continue forever. x=-6.
4. Multiplying a number by 10 shifts the decimal point one 10. Subtracting 6 from both sides of the equation gives
place to the right. So, 0.85 • 10=8.5. z+1 =-3.
1 10
Written as a mixed number, 8.5=8 2 .
Then, multiplying both sides by 10 gives
or z+1
10 =-3
We start by writing 0.85 as a fraction and simplifying. z+1
10 10=-3 10
• •
85 17
0.85= 100 =20 z+1=-30.
© 2017, AoPS Incorporated.
Are you ready for Beast Academy 5D?
Finally, subtracting 1 from both sides gives 2
We know that 5 • 5=2.
z=-31. 2
If we multiply 5 by any positive integer less than 5, then
11. We first translate the words into an equation. “Five times n” 2
the 5 in the denominator of 5 will not cancel and the result
is 5n, so “three more than 5 times n” is 5n+3. will not be an integer.
Therefore, the equation given by the sentence is 5n+3=7. So, 5 is the smallest positive integer we can multiply by
Subtracting 3 from both sides of this equation gives 0.4 to get an integer result.
5n=4. 16. The sign of the answer is determined by the number of
negatives in the product.
Dividing both sides by 5 gives n= 45 . You may have instead
4
written 5 as a decimal: 0.8 or .8. The negative numbers in this sequence are
-2, -4, -6, ..., -26, -28, -30.
12. In an arithmetic sequence, the same amount is always
added from one term to the next. The amount that is added To count the number of negatives in the list above, we
to get each next term is called the common difference. divide each of the numbers by -2. This gives us

The sequence has 1st term 39 and 4th term 53, so the 1, 2, 3, ..., 13, 14, 15.
53−39 14 2 Therefore, there are 15 negative numbers in this list, so
common difference is 3
= 3 =4 3 . We use this to fill
in the remaining blanks as shown. there are 15 negative numbers in the original sequence.
+4 23 +4 23 +4 23 +4 23 +4 23 +4 23 Any product with an odd number of negatives (and no
zeros) is negative.
39, 43 23 , 48 13 , 53, 57 23 , 62 13 , 67, ... 17. There are 20 numbers in the given product, each of which
contributes 1 factor of 5 to the prime factorization:
13. The sequence has 1st term 50 and 3rd term 12, so the
12−50 -38
common difference is 2 = 2 =-19. We use this to fill (1 • 5), (2 • 5), (3 • 5), ..., (18 • 5), (19 • 5), (20 • 5).
in the remaining blanks as shown. However, multiples of 25=52 have two 5’s in their prime
−19 −19 −19 −19 −19 −19 factorizations. So, these numbers contribute 2 factors of
5 to the prime factorization. There are four multiples of 25
50, 31, 12, -7, -26, -45, -64, ... in this product, which each contribute one more 5 to the
14. We consider the prime factorization of the expression on prime factorization: 25, 50, 75, and 100.
the left. 53=125 is greater than 100, so none of the numbers in
12 • 14 • 21 • 2=(2 • 2 • 3) • (2 • 7) • (3 • 7) • 2 the given product contribute 3 or more factors of 5 to the
=2 • 2 • 2 • 2 • 3 • 3 • 7 • 7. prime factorization.
We can split these prime factors into two equal groups: So, the exponent of 5 in the prime factorization of the
product is 20+4=24. Therefore, the power of 5 in the
2 • 2 • 2 • 2 • 3 • 3 • 7 • 7=(2 • 2 • 3 • 7)(2 • 2 • 3 • 7)
prime factorization of 5 • 10 • 15 • ... • 90 • 95 • 100 is 524.
= (2 • 2 • 3 • 7)2.
Since 2 • 2 • 3 • 7=84, we have 18. From the given ratio, we know that for some value of x,
2
Edgar is 3x years old and Frank is 6x years old.
12 • 14 • 21 • 2= 84 . The sum of their ages is 3x+6x=9x.
15. Multiplying 0.4 by 10 moves the decimal point in 0.4 one Since the sum of their ages is 36, we write an equation:
places to the right, giving the integer 4:
9x=36.
0.4 • 10=4. Dividing both sides by 9 gives x=4.
1
Both 10 and 4 are divisible by 2. So, multiplying 0.4 by 2 of Therefore, Edgar is 3 • 4=12 years old.
1
10 will give a result that is 2 of 4.
or
1 1
2
of 10 is 5, and 2 of 4 is 2. So, we have We first simplify the ratio:
0.4 • 5=2. 3:6=1:2.
5 and 2 are not both divisible by an integer greater than 1, Therefore, for some value of y, Edgar is y years old, and
so 5 is the smallest positive integer we can multiply by 0.4 Frank is 2y years old. The sum of their ages is y+2y=3y.
to get an integer result. Since the sum of their ages is 36, we write an equation:
or 3y=36.
We write this decimal as a fraction in simplest form, Dividing both sides by 3 gives y=12. Edgar is 12 years
4 2
0.4= 10= 5 . old.

© 2017, AoPS Incorporated.

You might also like